All+ORE+OSCEs.pdf

All ORE OSCEs September 2012 1 Actor (communication) OSCEs All OSCEs ................................................

Views 88 Downloads 1 File size 4MB

Report DMCA / Copyright

DOWNLOAD FILE

  • Author / Uploaded
  • Ana
Citation preview

All ORE OSCEs September 2012

1

Actor (communication) OSCEs All OSCEs ............................................................................................................................ 4 Actor OSCEs (communication) ................................................................................................ 6 1.

Consent for extraction of an upper 1st molar under local anaesthesia ........................ 7

2.

Consent for extraction of a lower 3rd molar ................................................................. 8

3.

Pain and anxiety control for nervous patient undergoing extraction.......................... 10

4.

Post-extraction instructions ....................................................................................... 13

5.

Dry socket - pain after extraction .............................................................................. 14

6.

Bleeding after extraction – patient on warfarin .......................................................... 16

7.

Chronic advanced periodontitis & need for extraction of all teeth ............................. 19

8.

Chronic advanced periodontitis & need for extraction of some teeth ........................ 20

9.

Dental treatment for a patient on bisphosphonates .................................................. 22

10.

Broken file in root canal ............................................................................................ 25

11.

Adult patient with impacted UL3 and retained & discoloured (ULC) ......................... 26

12.

Discoloured upper right central incisor ...................................................................... 27

13.

Missing tooth............................................................................................................. 29

14.

Erosion - causes and management .......................................................................... 31

15.

Angina pectoris – take history and give diagnosis .................................................... 32

16.

Oral Hygiene Instructions ......................................................................................... 34

17.

Recurrent aphthous mouth ulcers ............................................................................. 35

18.

Irreversible pulpitis .................................................................................................... 37

19.

Lip laceration by a colleague .................................................................................... 38

20.

Dietary advice and diet analysis ............................................................................... 39

21.

Bottle caries .............................................................................................................. 43

22.

Lichen planus/lichenoid reaction ............................................................................... 44

23.

Avulsed tooth – phone call ........................................................................................ 46

24.

Needle-stick injury .................................................................................................... 48

25.

TMJPDS ................................................................................................................... 50

Skilled OSCEs ....................................................................................................................... 53 Rubber dam ....................................................................................................................... 54 Hand washing .................................................................................................................... 60 Intramuscular injection ....................................................................................................... 54 Suture................................................................................................................................. 61 2

Blood pressure ................................................................................................................... 62 Denture faults ..................................................................................................................... 62 CPR.................................................................................................................................... 62 BPE .................................................................................................................................... 62 Dental charting ................................................................................................................... 62 IOTN classification ............................................................................................................. 62 Referral letter ..................................................................................................................... 62 Lab documents ................................................................................................................... 62 Radiograph shown and we have to write like multilocular etc ............................................ 62 Radiographic faults............................................................................................................. 62 Treatments for hygienists and dentists ............................................................................... 62 Arranging radiographic film ................................................................................................ 62 Tooth splint......................................................................................................................... 62 Denture candidosis (aetiology,characteristics,treatment and other relevant information) .. 62 Identifying surgical instruments .......................................................................................... 62 Waste disposal ................................................................................................................... 57 Cephalometric analysis ...................................................................................................... 62 Plaque Control Record (the O'Leary Index) ....................................................................... 63 CPITN-E probe ............................................................................................................... 65 RPD design ........................................................................................................................ 73 Surveyor ............................................................................................................................. 76 Written OSCEs ...................................................................................................................... 81 Prescription writing ............................................................................................................. 82 Inhalation Sedation Machine .............................................................................................. 87 IOTN................................................................................................................................... 98 Lichen planus and lichenoid reaction ............................................................................... 103 Denture stomatitis ............................................................................................................ 105 Referral letter ................................................................................................................... 107 Denture faults ................................................................................................................... 113

3

All OSCEs Actor OSCEs (communication) 1. Consent for extraction of an upper 1st molar under local anaesthesia 2. Consent for extraction of a lower 3rd molar 3. Pain and anxiety control for nervous patient undergoing extraction 4. Post-extraction instructions 5. Dry socket - pain after extraction 6. Bleeding after extraction – patient on warfarin 7. Chronic advanced periodontitis & need for extraction of all teeth 8. Chronic advanced periodontitis & need for extraction of some teeth 9. Dental treatment for a patient on bisphosphonates 10. Broken file in root canal 11. Adult patient with impacted UL3 and retained & discoloured (ULC) 12. Discoloured upper right central incisor 13. Missing tooth 14. Erosion - causes and management 15. Angina pectoris – take history and give diagnosis 16. Oral Hygiene Instructions 17. Recurrent aphthous mouth ulcers 18. Irreversible pulpitis 19. Lip laceration by a colleague 20. Dietary advice and diet analysis 21. Bottle caries 22. Lichen planus/lichenoid reaction 23. Avulsed tooth – phone call 24. Needle-stick injury 25. TMJPDS Skilled OSCEs 26. Rubber dam 27. Hand washing 28. Intramuscular injection 29. Suture 30. Blood pressure 31. CPR 32. BPE 33. Dental charting 34. Lab documents 35. Radiograph shown and we have to write like multilocular etc 36. Radiographic faults 37. Treatments for hygienists and dentists 38. Arranging radiographic film 39. Tooth splint 40. Denture candidosis (aetiology,characteristics,treatment and other relevant information) 41. Identifying surgical instruments 42. Waste disposal 43. Cephalometric analysis 44. Plaque Control Record (the O'Leary Index) 45. RPD design 46. Surveyor Written OSCEs

4

47. Prescription writing 48. Inhalation Sedation Machine 49. IOTN 50. Lichen planus and lichenoid reaction 51. Denture stomatitis 52. Referral letter 53. Denture faults

5

Actor OSCEs (communication)

6

1.

Consent for extraction of an upper 1st molar under local anaesthesia

Use patient’s name and communicate empathically and avoid jargon

Good Morning Mrs Jones, I’m ________, one of the dentists here today and I’ve been given a task  to explain you about the removal of one of your upper teeth which has been troubling you. As you know, this tooth is damaged to a point beyond repair so the best is to have it out. I understand that you may be a little worried but I can assure you we’ll be very gentle during the whole procedure.  Explain surgical method

First, I will apply an anaesthetic cream on the gum to make it a little numb prior to the anaesthesia, so it is more comfortable for you. We will then make the tooth numb in which you may feel a sharp scratch. This procedure takes only a few seconds and will make your tooth and the area around it numb, so you will not feel pain. I will be very gentle throughout and if you want me to stop at any time you just need to lift your left arm and I will stop. We will then gently remove your tooth using some special instruments. As I said, there should be no pain, however, you will feel some pushing and pressure and there will be some bleeding. Please inform us if there is any pain and we will immediately stop the procedure and apply some more anaesthetic to numb the area. Complications

The tooth may come out in one piece but there is a small chance that it might break. If this happens and might have to cut the tooth in half or do a small surgery which involves a cut in the gum and removing a bit of bone around the tooth to pull it out. However, there should not be any pain as the area will be numb. Sometimes a bit of the bone behind the tooth may fracture. We call this tuberosity fracture. If this happens we will place some sutures and may also have to stabilise the area and ask you to be extra cautious and to come return for follow-up visits. I must also warn you that, as you can see in the X-ray, there is a small air space in the upper jaw which is close to the root of this tooth. There is a small chance of a communication to occur between your mouth and this area when the tooth is removed. If it is a small opening, it may close on its own, but if it is big, we’ll have to close it by stitching the gum together on top of this opening. We’ll also  give you some medicines, inhalations and nose drops for you to use and you’ll have to avoid  blowing your nose for 2 weeks. Final considerations

We will then close the wound with stitches which will be removed after a week and we will place a pack of gauze on the wound and you have to gently bite on it for half an hour to stop the bleeding. If there is bleeding after that, please contact us immediately. After the extraction, the area may become sore and swollen and there might be some jaw ache and discomfort on eating for a few days after removal of the tooth so we will give you pain killers. You might also have slight difficulty in opening the mouth for a few days and you might need to take a day or two off work to recovery properly. To guarantee good healing, you must to be careful not to disturb the wound because it will be quite fragile. Avoid touching it with you hand or tongue. You’ll need to eat soft and cold foods such as  mash potatoes or scrambled eggs for rest of the day. Ice cream is also a good alternative. We’ll explain you about care of your wound and also give this information in writing. Do you have any questions? May I have your permission to go ahead with the treatment? Thank you. If you are anxious about this procedure, please don’t worry because there are methods to reduce your  anxiety and I can explain them to you in more detail later.

7

2.

Consent for extraction of a lower 3rd molar

Use patient’s name, communicate empathically, and avoid jargon

Good Morning Mrs Jones, I’m _______, one of the dentists here today and I would like to talk you  through the removal of the lower tooth which has been troubling you. I can understand that you are a little worried but I can assure you we’ll be very gentle with the whole procedure. Explain reason for surgery

As you know, this tooth hasn’t come out straight. It is impacted and has caused problems like pain  and swelling a few times. According to official guidelines from the National Institute of Clinical Excellence (NICE), when this problem happens more than once then the tooth is best removed to avoid further complications as they tend to become worse and more frequent. Explain surgical method

First, we will make the tooth numb in which you may feel a sharp scratch. You will then feel tingling on one side of your tongue and lower lip and then the area and the tooth will go numb. We’ll then remove the tooth with a minor surgery which involves a cut in the gum and removing a bit  of bone around the tooth to pull it out. You will not feel any pain but you may feel some pushing and pressure and there will be some bleeding. We will then close the wound with stitches which will be removed after a week. We will also place a gauze pack inside your mouth to stop bleeding on which you will have to bite on for half an hour. If there is bleeding after that, please call us immediately. Post- surgical instructions

After the procedure, the area will be sore and swollen for a few days but we will give you some pain killers. You might also have some difficulty opening the mouth for a few days, and you might have to take a few days off work. You have to be careful not to bite on your numb lip and avoid disturbing the wound coz it will be quiet fragile. Possible complications

As the roots of this tooth are close to an important nerve called the Inferior Dental Nerve, there is a small chance of an injury to the nerve and you might get numbness or tingling on that side which usually lasts a few days. The probability of this to happen is of 10-20% but there is also a very small chance (of less than 1%) of it being permanent. There is also a chance of injury to a nerve of your tongue called Lingual Nerve, which can cause some alteration in taste with 2% temporary and 0.5% to be permanent. We will be very careful; however, as we have to work on a small space, there is also a possibility of adjacent teeth to be damaged during the surgery. As I said, we will give you pain killers for a few days to reduce the pain you might feel after the surgery and, if there is any chance of further infection, then we might prescribe you antibiotics. We’ll give you all this information and all the aftercare instructions of your wound in writing. Do you have any questions? If you’ve understood me, may I have your permission to go ahead with the treatment? Thank you. If you are anxious about this procedure, please don’t worry because there are methods to reduce your anxiety and I can explain them to you in more detail later.

8

Candidate instructions Mr/Miss Jones has had 3 episodes of pericoronitis associated with her lower left third molar, two of which needed antibiotic prescription. You are planning to remove her mesially impacted and partially erupted lower left third molar under local anaesthetic. On radiograph the roots of the tooth are in close relationship to the inferior dental nerve canal. Please consent this patient to have the above mentioned procedure. Patient brief You are Miss Jones, 33, attending you dentist to seek explanation as to how your dentist will remove your wisdom teeth and what problems may arise after the surgery. You would like to know an outline of the surgery itself, and also what problems will there be after the surgery. It is important that the dentist makes you aware not only of the pain and swelling that will follow but also the risk of damage to the inferior dental nerve and lingual nerve. If he does mention this then you should try and establish how long the effects will last, where will the numbness be most noticeable. You are medically healthy, with no allergies and you are not taking any other medication.

9

3.

Pain and anxiety control for nervous patient undergoing extraction.

Use patient’s name, communicate empathically, and avoid jargon

Good Morning Mr Jones, I’m ________, one of the dentists here today.  I understand that you are a  little worried and anxious about the removal of your tooth so I would like to explain to you some methods we can use to reduce your anxiety and make it more comfortable for you. Explore why patient is requesting a general anaesthetic. Sympathise and reassure. Advise patient that the tooth is mobile and can be taken out under local anaesthetic safely and quickly in the practice. Sedation and GA will require referral which may delay treatment Explain Sedations

To help you relax during the procedure we can give you some medication which makes you feel very comfortable. We would still need to make the tooth numb. However, you will be much more relaxed. This is called sedation and, in your case, can be done in two different ways: Oral sedation

In the first method, we can give you a sedative medication in the form of tablet about an hour before the surgery, this is called; oral sedation. The medication takes a while to get into your system and then you’ll feel sleepy for about 6 hours. This method is very simple; however, as people respond  differently to the drug, it is not so reliable. It is suitable for mild anxiety and restorative procedures but unlikely to be sufficient for surgical extractions. It may be used in addition to other techniques if the patient is so anxious that they may not even attend for the appointment. Intravenous sedation

An alternative to this is method is to deliver the medication straight into your blood stream through a fine plastic tube into a vein on your arm or hand. This is called intravenous sedation and you will need to fast for 2 hours prior to the procedure. You’ll be awake during the procedure and will be able to talk and follow instructions; however, you’ll tend to forget the experience. This method requires an especially trained dental team but is relatively simple can be administered in a dental practice setting. Post-operative instructions

You have to be careful afterwards because this drowsiness might last for the whole day, so please avoid driving, using public transport, operating machinery, drinking alcohol or signing important papers on that day. You will also need to bring a responsible person to accompany you to the dental surgery and to take care and stay with you on that day. Unfortunately, none of these methods can be used if you have a breathing problem such as asthma or even common cold because the drugs involved tend to reduce your breathing ability. General anaesthesia

Another option is what we call general anaesthesia, which is done in the hospital. You will need o fast for 6 hours before this procedure. You will be given some drugs by a specialist and you’ll be  completely asleep and will not feel anything. You will not be able to breathe on your own so a device will be attached to your throat to help you breathe. The drowsiness after the procedure may last for a few days and you’ll have to take a few days off work. You may also feel a little unwell and might  vomit after the procedure. Before you can have it done, you will need to be assessed by a doctor for a full check-up in a pre-assessment appointment. There are also risks associated to this procedure. As you won't be able to breathe on your own there is an increased risk of chest infection or respiratory issues post-operatively due to history of smoking and obesity. There may also be potential airway issues as the patient is obese as well as an increased risk of hospital acquired infections. I must also mention that there is a risk of death of about 1/100,000 and this risk is increased amongst smokers, and people suffering from diabetes, respiratory problems, and obesity. The drugs involved are not as safe as sedation, so it is better to avoid it, unless completely necessary. The whole procedure lasts for a few hours and you’ll probably get to leave hospital on the same day. Do you have any questions? Thank you

10

11

12

4.

Post-extraction instructions

Use patient’s name, communicate empathically, and avoid jargon

Good morning Mr James, I am ________, your dentist today, I’ve been told that your tooth has  been removed or has to be removed (according to the question) and I’m here to explain to you how  to take care of this wound. Is that alright? Explain post-extraction precautions

After the tooth is removed, we place a piece of gauze on top of the wound and you’ll have to bite on  it for half an hour to stop the bleeding. If the bleeding continues after you have left the surgery, please don’t panic, because most of the  liquid is saliva which might be mixed with a little bit of blood. If you feel that the bleeding is more, fold a clean handkerchief and place it over the wound and bite firmly and keep it pressed for 15 -30 minutes. If the bleeding still continues please call to the surgery or the local emergency service or come back to the surgery immediately. The wound is quite fragile and it gets disturbed easily in which case it might bleed again and takes longer to heal. So, avoid touching it with your tongue or finger, avoid eating on this side for few days and do not spit or rinse your mouth for the rest of the day. Brush your teeth gently for a week, it might be uncomfortable to brush around the wound, so don’t  brush that area for a day. Avoid strenuous exercises, hot fluids, alcohol, hard or chewy foods for rest of the day. So you may have to eat soft and cold food such as mashed potatoes or scrambled eggs for rest of the day. Ice cream is also allowed. If you are a smoker, avoid smoking for at least 24 hours as smoking delays the wound healing. Avoid heavy exercise or work for at least one or two days. The wound might be sore and slightly painful for a few days which is normal, so I’ll prescribe you  some pain killers like Paracetamol which you can take 2 tablets every 4 hours as required. If the pain is severe, contact to the surgery. You might also have some swelling and some difficulty in opening the mouth and it takes a few days to reduce. You may initially have some discomfort while eating but all these problems should go away in a week. The area around your tooth/lips, cheek and tongue may be numb for about 2 hours after tooth removal, so take the pain killers within 2 hours, and please be careful not to bite your lip or tongue while it is numb. If the pain increases or gets worse after a few days, please come to the surgery. After 24 hours, gently rinse the wound with warm salty water for about 15-20 seconds three times daily after food for a few days. This helps healing and keeping the area clean. We will also give all these instructions in writing. Do you have any questions? Thank you.

13

5.

Dry socket - pain after extraction

Explain condition, causes and management. The patient keeps saying she is in pain and asking when the pain was going to wear off. Use patient’s name, communicate empathically, and avoid jargon

Good morning Mrs Jones, I’m ________, one of the dentists here today. May I know how can I help you today? It is unfortunate that you have this problem. I assure that I can do something about it. Before that, I would like to know few details. Is that alright? Take history of presenting complaint

Where do you feel the pain? Which tooth was removed? Was there any difficulty in removing the tooth? When the tooth was removed and when did this pain start? Did you follow all the instructions given after the removal of the tooth such as not to rinse vigorously and not to smoke for a week? How strong is the pain? Can you describe this pain to me? (Throbbing and aching) Is the pain continuous or does it keep coming and going? (Constant pain) Is there anything which increases or reduces the pain? Does the pain spread anywhere else? Has the pain been increasing? Have you also noticed bad breath or bad taste? Does it disturb your sleep? (Cannot sleep) Have you tried any pain killers? (No good) Or any other treatments? Have you had a tooth removed before, if yes, did you have a similar problem? Take brief medical history

Are you taking any medications for any other problems or have any major health concerns like diabetes, allergies, etc. Are you taking any contraceptive pills? Do you smoke? When did you smoke after the tooth was removed? Diagnosis and basic explanation of disease and aetiology - dry socket (alveolar osteitis)

From what we have discussed, it seems that you have a problem after removal of teeth which we call dry socket. It happens in about 3-20 % of all extractions. It is more common in lower teeth, women, smokers and in difficult extractions or if any previous history. Normally when a tooth is removed, blood fills up in the empty space and then forms a solid mass, the blood clot, on which the healing takes place. If due to any reason such as smoking, taking the pill or injury to this area, this mass breaks or gets removed, the healing slows down and the bone below it is exposed to the open. This bone is very sensitive which causes the pain and then food gets lodged and some bugs build up in this area and you start getting bad breath as well. This condition is very painful but it does not have any serious consequences, the area will heal slowly but it takes more time. What I can do today is, numb this area with an injection and clean and wash it with an antiseptic containing Chlorhexidine. I’ll then place a dressing called Alvogyl paste in this space which will relieve the pain. This dressing gets dissolved by itself. I’ll also give you pain killers to ease the pain at home. We’ll repeat this every other day until the pain is completely relieved, hopefully in 2 or 3 sittings. After that you can take care of it at home by rinsing with a mouthwash or warm salt water rinses. I would also advise you to avoid smoking for a few days. Thanks you. Do you have any questions?

14

Candidate instructions Mrs Jones is a 65 year old lady who has attended your surgery in pain. You removed her UR4, three days ago and since then she has been in a great deal of pain. It started within 24 hours of the extraction and is a continuous ache which is not helped by any pain killer she has at home. She can now see some tooth in the socket and is annoyed that you have left some of the tooth behind. She also says that her daughter commented that her breath now smells. You have the picture of the patient's mouth to help you make the diagnosis. Speak to the patient to explain what is happening and what you will do for her?

15

6.

Bleeding after extraction – patient on warfarin

Use patient’s name, communicate empathically, and avoid jargon

Good morning Mr Jones, I’m _______, one of the dentists here today. How can I help you? Mr Jones I understand you are worried and concerned and I will do my best to help you. I would like to know few details of the problem. Is that ok? Take history of presenting complaint

When was the tooth removed? When did the bleeding start? Was the extraction difficult or did it take a long time? Were there any stitches given? Did you do anything to stop the bleeding? Did you follow the instructions given by your dentist for the care of the wound? Is it bleeding a lot? Did it stop for some time or has it been continuous? Take brief medical history

Do you have any health problems? Are you taking any drugs or medication, prescribed or over the counter? How long have you been taking this medication (Warfarin or ibuprofen) and what is it for? Do you have any warning cards or a yellow booklet? Can I have a look at it? Was any blood tests done on the day that the tooth was removed? Was your INR checked before extraction? Do you know the value? Does your INR fluctuate? Do you have any other diseases such as high blood pressure or liver or any bleeding problems? Did you take any pain killers after the tooth was removed? (Ibuprofen) Diagnose, briefly explain problem and aetiology

Mr Jones, the medicine that you are taking (Warfarin) is given to make your blood thin so that it can flow smoothly in your body, but this may also cause wounds to bleed for longer. We check this increased bleeding by a measure called the INR which is updated on your yellow warning card regularly. In your case, the value was safe enough, but there may still be more bleeding compared to a normal person. Another reason why the bleeding is prolonged is because you were taking Ibuprofen along with this Warfarin, which is known to increase the effect of Warfarin. You also failed to follow the post-extraction instructions and smoked and rinsed your mouth which made loss of blood clot or may have delayed its formation. Explain treatment plan to stop the bleeding

Don’t worry, we will take care of this and the bleeding will stop. Most of the blood you see might be a little blood mixed with a lot of saliva, so maybe the actual loss of blood might be much less than what it appears to be. First I’ll check your mouth,clean it and identify where exactly the blood is coming from. I’ll then apply  some pressure with my fingers to bring the walls of the wound closer and place some stitches on top of it then ask you to bite on a gauze pack for half an hour. The bleeding should then stop Once the bleeding stops, we’ll wait for an hour until you are safe to go home and I’ll explain to you  how you need to take care of the wound. If the bleeding still doesn’t stop after an hour, I’ll give you some medication and I might also have to refer you to the hospital. I may also need to consult your doctor. Thank you Mr Jones. Do you have any questions?

16

Candidate instruction You are an emergency dentist attending Mr Jones in your practice who had an extraction of UR6 carried out at another dental practice 9 hours ago. He presents with persistent bleeding from the socket. Please take appropriate history to determine the cause of the bleeding. Actor's props Anticoagulant clinic record book (yellow book), box of propranolol, box of warfarin, box of ibuprofen, gauze pack, written instructions after extraction, paper work (consent, ITA) Actor's (patient) information: Full Name: Mr Victor Jones DOB: 22/11/44 (66 years-old) Address: 1 Lame Street, Palmers Green NZ11XC Dental history: Brush twice daily (afternoon and before going to bed) Flosses every evening at 10pm, visits dentist every 6months Diet: Low sugar (1 tsp in a cup of tea twice daily) Last dental treatment: extraction of an unrestorable tooth UR6 Doesn't wear denture. Never had filling or extraction before in life. Happy with smile & appearance of teeth. Medical history: Atrial fibrillation, mild osteoarthritis, hypertension (well controlled), visit GP for repeat prescription only when necessary. Allergic to shell fish (rash) and taking warfarin 5mgs (INR 2-3), propanolol 10mgs, ibuprofen 200mgs Visit anti-coagulant clinic every week (last visit was yesterday and INR was 2.9). Family history: nil Social history Marital status: Widow with no children and lives alone in a flat. Smokes 5-10/day for the past 35 years and is not interested in quitting. Alcohol: teetotal Occupation: Retired Civil Worker Patient instruction You are the above named patient and had an extraction done 9 hours ago at another dentist. Since the extraction you have been bleeding (oozing out of the socket) but with no pain. You have tried biting on gauze for 30 mins and did it twice but in vain. You are very annoyed and frustrated. You have already smoked 2 cigarettes, 4 hours after the extraction and rinsed your mouth thrice with cold water to get rid of the bad taste inside the mouth. You have been taking Ibuprofen for the last 4 days as your joints are hurting. The dentist gave you the written and verbal instructions and as far as you know you tried following them. Your INR was taken yesterday afternoon and it was 2.9.Usually your INR stays within the range of 2-3. You couldn’t eat anything since then and you are concerned about the blood loss and would like to know the likely cause of the bleeding. Only give the yellow book and list/boxes of medication if candidate asks for it. You will also have some paper work & gauze, just give everything with above.

17

Some people will start talking about the treatment but you may need to drag them back to the cause of the bleeding as the questions specifically ask for finding out the cause of the bleeding socket. You need to ask them: Why am I bleeding continuously? Is it because of rinsing/smoking that I am bleeding? Am I bleeding because of warfarin? Am I bleeding because of Ibuprofen? Have I lost a lot of blood? Do I need to stop taking warfarin/ Ibuprofen?

18

7.

Chronic advanced periodontitis & need for extraction of all teeth

Use patient’s name, communicate empathically, and avoid jargon

Good morning Mr Jones, my name is _______, one of the dentists here today. How may I help you? Take history of presenting complaint and a brief medical and social history

When did you first notice it? Is the mobility increasing? Did you consult your dentist? Is there anyone else in your family with a similar problem? Do you suffer from any major disease like diabetes or blood disorders? Do you smoke? How many cigarettes a day? For how long? Have you ever smoked or are trying to quit at the minute? Are you under a lot of stress in your life at the moment? Explain causes and risk factors –patient smokes

After seeing your reports, I’ve noticed that your gums and bone around your teeth are coming down and slowly losing the grip on your teeth and the teeth have become loose. It seems like you have a long standing gum disease which has progressed slowly over many years. We call it chronic periodontitis. There can be many factors that contribute this to happen such as: genetic factors, smoking, stress, diabetes and poor oral hygiene. These factors increase the speed of the destruction of bone support of your teeth. There will also be attachment loss between your gums and teeth. This can lead to some pockets to form and bacteria can trap in those and cause more bone loss. Smokers have more bugs that cause this problem and it also reduces the ability of the mouth to protect the gums from attack of these bugs. So stopping smoking is very important. In diabetes, the ability of the body to respond to the attack of these bugs is also reduced. Explain treatment

Unfortunately, in your case, as you can see in this X-ray, the disease has progressed to a level where your remaining teeth cannot be saved. This is really sad and it might come as a shock to you that we will have to remove those teeth because if we don’t do that, the bone might be lost even more and we might not even have enough  bone to support a denture properly. These teeth will be lost anyway, but at least we’ll be able to  save the bone. You can take your time to think. We’ll numb the teeth and initially remove some of the most badly affected ones. We will give you dentures immediately after the extraction, so you are not left without teeth. We can add new artificial teeth to this denture as we remove more teeth. These dentures will be temporary and will need regular adjustments on many appointments. We can give you a permanent set of dentures later on, after complete healing takes place. Do you have any questions? Thank you.

19

8.

Chronic advanced periodontitis & need for extraction of some teeth

Use patient’s name, communicate empathically, and avoid jargon

Good morning Mr Jones, my name is _______, one of the dentists here today. How may I help you? Take history of presenting complaint and a brief medical and social history

When did you first notice it? Did you consult your dentist for it? Is the mobility increasing? Is there anyone else in your family with a similar problem? Do you suffer from any major disease like diabetes or blood disorders? Do you smoke? How many cigarettes a day? For how long? Have you ever smoked or are trying to quit at the minute? Are you under a lot of stress in your life at the moment? Explain causes and risk factors –patient smokes

After seeing your reports, I’ve noticed that your gums and bone around your teeth are coming down and slowly losing the grip on your teeth and the teeth have become loose. It seems like you have a long standing gum disease which has progressed slowly over many years and it is more pronounced in the back teeth. We call it chronic periodontitis. There can be many factors that contribute this to happen such as: genetic factors, smoking, stress, diabetes and poor oral hygiene. These factors increase the speed of the destruction of bone support of your teeth. There will also be attachment loss between your gums and teeth. This can lead to some pockets to form and bacteria can trap in those and cause more bone loss. The plaque formed in smokers has more bugs causing this problem and it also reduces the ability of the mouth to protect the gums from attack of these bugs. So stopping smoking is very important. In diabetes, the ability of the body to respond to the attack of these bugs is also reduced. Explain short term and long term treatment

Unfortunately, in your case, as you can see in this X-ray, the disease has progressed to a level where some of your remaining teeth cannot be saved. This condition is affecting some teeth more than others and if we remove the ones that are most affected, we can then focus on saving the ones that still have some good bone support. If we don’t  do that, the bone might be lost even more, to a level that there may not be enough bone to support a partial denture, used to replace the teeth you will loose. For you to be able to keep your remaining teeth for as long as possible, you’ll need to make some  changes to the way you look after them. It is important to use a soft toothbrush and to change it every other month. Electric toothbrushes are also very good. You’ll also need to clean in between  your teeth either with dental floss or with interdental brushes or picks. In the process of recovering, your gums might come down a bit and this may cause your teeth to get a little sensitive. This is normal and if it happens, please let us know so we can apply a protective coating on them. We might also prescribe you some mouth rinses for a few days. The remaining teeth will be very important for you, as you can continue to use them for chewing. They may also be helpful to stabilise future plastic or metal dentures or even support bridges. You will need regular professional cleaning of your teeth so we can clean the plaque below the gum level and also remove the hard tartar that accumulate on your teeth. Initially, I would like you to come to see us every 3 months until we are able to control your condition. If we see that you are recovering well, the condition is stable and you are able to keep the area clean, we may suggest you to come every 6 months instead. However, if we see that it is actually getting worse, even after all our efforts; we may refer you to a specialist who is more able to deal with more complex cases. Do you have any questions? Thank you.

20

Chronic periodontitis Candidate instructions Mr Chohan is a 40 year-old smoker who presents with recurrent soreness of his gums and occasional pain on biting from several teeth which has become worse over the last few years. His radiographs are displayed below along with BPE scores. BPE: 434/234*

Please explain the radiographic and BPE findings to the patient and explain the nature and treatment of periodontal disease. Discuss with the patient the long-term indications of his disease and possible further treatment needs.

21

9.

Dental treatment for a patient on bisphosphonates

Use patient’s name, communicate empathically, and avoid jargon

Good morning Mr Jones, my name is _______, one of the dentists here today. How may I help you? Take history of presenting complaint (pain history) and a brief medical and social history

Are you currently taking medications, either prescribed or over the counter? o Patient says he is on bisphosphonate. Find out what condition, for how long they have been taking the bisphosphonates and the route of administration? Do you have diabetes or suffer from high blood pressure? Do you smoke or have recently stopped? Give diagnosis and explain findings

Mr Smith, Based on the information provided by you, I gather that you have a tooth which was root canal treated but is now painful. The reason for this pain is that there is some decay formed under the cap which covers your tooth. Also, the root canal filling seems to be a bit short and has not worked. This leakage under the cap may have allowed germs to gain entry and to travel to the root tip of the tooth and cause an infection. This may happen in a few cases and it is difficult to predict. Explain complications associated with bisphosphonates

I understand you want the tooth taken out. However, as you take bisphosphonates I must warn you of possible complications and inform you alternative options. Bisphosphonates reduce the breakdown and loss of bone. They are also used to fight tumours. However, they affect the capacity of the bone to heal and may cause death of part of the jaw bone. This is a condition which we call osteonecrosis of the jaw (ONJ). The risk of developing it varies according to the strength and the length of time the drug has been taken. If it is taken in the form of tablets (low dose) the risk is 1/100,000. However, in your case, as you are taking it through injections (higher dose) the risk is 10 times higher 1/10,000. The risk is also increased in the bottom jaw when compared to the top jaw and also amongst smokers and people with diabetes. Stopping the medication does not decrease the risk as it stays in the system for a very long time. Another important point to consider is that the medical benefits of taking bisphosphonates far outweigh risks. If we remove the tooth we might be unnecessarily triggering bone death and cause you trouble. Present alternative option (reRCT), advantages and disadvantages

However, there is an alternative treatment; I would like to suggest to take the crown off to enable me see the root properly and if possible have the root cleaned and sealed again (re-RCT). The advantages are that you keep the tooth and avoid bone death. However, re-treating the tooth decreases the success of treatment from 85 to 60%. This may take about 3 visits after which I will place a new crown on it. If you still prefer the tooth to be taken out, I may have to seek an opinion from a specialist and find out whether I can treat you here or whether I should refer you to a specialised service. To reduce the chance of you facing this situation again I would like you to maintain a good oral hygiene and come here more often for regularly check-ups. If you experience loose teeth, pain or swelling in your mouth while using bisphosphonates (these may be symptoms of ONJ), please tell your dentist and doctor about this as soon as possible. Do you have any questions? Thank you. Oral bisphosphonates Intravenous/injected bisphosphonates Alendronate(Fosamax, Fosavance) Ibandronate (Bondronat, Bonviva) Sodium clodronate (Bonefos, Loron) Disodium pamidronate (Aredia) Disodium etidronate (Didronel, Didronel PMO) Zoledronic acid (Aclasta, Zometa) Ibandronate (Bondronat, Bonviva) Risedronate sodium (Actonel, Actonel Once a Week) Disodium tiludronate (Skelid)

22

Question 1 Mr/Mrs Green attends your dental surgery taking Bisphosphonates and requires extraction of a lower second molar. What further information you need to obtain from him. Please take the appropriate history, explain him/her the concerns and complications of having a dental extraction under this circumstances. Question 2 Leaky margins of crown on molar with RCT done but with periapical radiolucency. Patient on IV bisphosphonates. Explain treatment options and associated risks. Question 3 (RCS) A 75 year old patient of yours (Mr/Mrs Scholes) attends for an appointment relating to his/her LR6. The tooth was restored with an MOD direct composite restoration 5 years ago and has fractured a cusp approximatelly 6 months ago. In the last few days the tooth has started to cause a spontaneous, long-lasting pain which has kept the patient awake at night. The patient also has: Osteoporosis for which he/she has been taking bisphosphonate (Fosamax 10mg) orally once daily since 2000. The patient has no allergies or any other relevant medical condition. The LR6 has a large MOD composite restoration and a fractured mesio-lingual cusp. It is not tender to pressure and responds negatively to warm gutta percha, ethyl chloride spray and electric pulp testing. The patient keeps an excellent oral hygiene, has no periodontal disease and has a mildly restored dentition. Please provide this patient with treatment options followed with their advantages, disadvantages and also risks.

1 2

3 4 5 6 7 8 9

Introduces themselves (name and assignation) and uses patient’s name Takes a concise history - Type of Bisphosphonate - Via of administration - Time of taking it - Reason of taking it Establishes patient is on osteonecrosis of the bone risk Establishes and discusses the risks of doing extraction Invites questions from patient and answers them with clinical knowledge Gives explanation of procedure and post-operative instructions Advises referral to hospital if it is a complex extraction and communication with GP Shows empathy to patient and thanks patient Actor’s mark

10 Was the candidate able to produce sympathetic relationship with you? 11 Did you feel that you understood the explanation/information given? 12 Did the candidate address your anxieties properly?

done 2 Marks

Not done

2 Marks

1 mark 1 mark 1 marks 1 Mark 1 Mark 1 Mark (good: 3, Satisfactory: 2,Partially: 1, Poor: 0) 3 Marks 3 Marks 3 Marks

Management of BONJ

23

Avoid extractions or any oral surgery or procedures which may impact on bone (i.e. dento-alveolar, periodontal, periapical, deep root planing, complex restorations, implants) if there is an alternative treatment option. An exception is to consider removal of teeth of poor prognosis if this will avoid extractions or other bone impacting treatments later during the patient’s bisphosphonate therapy. In these  circumstances, follow the risk assessment and management recommendations below. If any extraction or any oral surgery or procedure which may impact on bone is necessary, assess whether the patient is at low or higher risk of BONJ as follows: the patient is at low risk before they have started taking bisphosphonates for any condition, or are taking bisphosphonates for the prevention or management of osteoporosis. the patient is at higher risk if any of the following factors is present: previous diagnosis of BONJ; taking a bisphosphonate as part of the management of a malignant condition; other non-malignant systemic condition affecting bone (e.g. Paget’s disease); under the care of a specialist for a rare medical condition (e.g. osteogenesis imperfecta); concurrent use of systemic corticosteroids or other immunosuppressants; coagulopathy, chemotherapy or radiotherapy. Advise the patient that there may be BONJ risk to enable informed consent, but ensure that they understand that it is an extremely rare condition. It is very important that a patient is not discouraged from taking medication or undergoing dental treatment. Record that this advice has been given. Note: There is no supporting evidence that BONJ risk will be reduced if the patient temporarily, or even permanently, stops taking bisphosphonates prior to invasive dental procedures since the drugs may persist in the skeletal tissue for years. If a patient has taken bisphosphonates in the past but is no longer taking them for whatever reason (i.e. completed or discontinued the course, taking a drug holiday), allocate them to a risk group as if they are still taking them. Management of low risk patients When other treatment options are not feasible, perform extractions, oral surgery, or procedures that may impact on bone as ‘atraumatically’ as possible; avoid raising flaps; achieve good haemostasis. Straightforward extractions and other bone impacting treatments can and should be carried out in primary care. The circumstances for seeking advice from an oral surgery/ oral and maxillofacial surgery specialist are the same as for a patient who is not on a bisphosphonate. After carrying out any invasive treatment, review healing at 4 weeks. If surgery sites fail to heal within 4 to 6 weeks, refer to an oral surgery/oral and maxillofacial surgery specialist. Note: There is no evidence supporting antibiotic or topical antiseptic prophylaxis in reducing the risk of BONJ. Management of high risk patients Contact an oral surgery/oral and maxillofacial surgery specialist to determine whether the patient should continue to be treated in primary care for any extraction or any oral surgery or procedure that may impact on bone, or whether referral is appropriate. When seeking this advice, include full details of the patient’s medical and dental history, and  preferably do so by letter.

24

10. Broken file in root canal Use patient’s name, communicate empathically, and avoid jargon

Good afternoon, Mrs Black, my name is ______________ and I am covering for your dentist who is on maternity leave. I am meant to be finishing the RCT today. Is that alright? Break the bad news about broken file left in the root canal and show the broken file on X-ray to patient

I had a look at the last X-ray taken of your tooth and I am sorry to tell you that a piece of instrument used to clean the root canals of your tooth was broken and is lodged inside your tooth. (Show the patient the broken file on the X-ray). There is always a little chance of this to happen during this type of procedure. I would like to apologise on behalf of my colleague and I will try to find out why you were not informed earlier. N.B: If the patient is worried about possible harmful effects such as cancer, poisoning and infection: Reassure the patient by saying: ‘’I understand your concerns but there is really no  need to worry about cancer or poisoning as there is really no connection between the two. However, in case we are not able to remove the broken bit, infection and pain could be a possible complication but there are ways of minimising this outcome. Explain treatment

The treatment options available for you are: I can try to remove the broken piece of instrument bypassing another instrument that can be used to pull it out and then, I can continue the RCT as usual. If this is not possible, I may use some vibrating instruments to remove it and then, I can continue the RCT as usual. If I am not able to remove the broken instrument by these methods, I may refer you to a specialist for the removal of the instrument or alternative treatments. He can try several specialised methods of removal and if he is successful, he will complete the treatment. If he doesn’t manage to remove it, he may fill the canal up to the level of the blockage and keep monitoring the tooth. He may also suggest doing a small surgery to clean and treat the tip of the tooth inside the bone. This is done in case the treatment does not work. However, I must tell you that referral to a specialist does not guarantee the success of treatment. The last option is removal of the tooth itself and its replacement with an artificial tooth. (Patient selects to have the treatment completed by dentist) Warn patient about the possible complication

The success rate of root canal treatment is usually 90%. However, in about 10% of cases, complications can occur, during or after root canal treatment, which can lead to failure. In cases where we are leaving the instrument behind, there can be pain, swelling or infection as the canal may not be cleaned in that area. Do you have any questions? I’ll make a note of this event and the explanation given to you in your records. Ok? Thank you Mr Black.

25

11.

Adult patient with impacted UL3 and retained & discoloured (ULC)

OPG and upper occlusal were given. Patient is concerned about what will happen? Will it erupt if left alone? What will bring it down? If so how? What are the options if milk tooth is extracted? Use patient’s name, communicate empathically, and avoid jargon

Good morning Mr Smith, my name is _____, I’m one of the dentists here today. How can I help you? I can understand that this is a front tooth and you are concerned about your appearance. I’ll need to  ask you some questions to understand the problem a bit better. Is it all right? History of presenting complaint (pain history) and a brief medical and social history

When did you notice that this tooth is discoloured? Is this (milk) tooth loose? Have you had any pain around this tooth? Have you consulted any other dentist for this problem? Explain the retention of the deciduous tooth and impaction of permanent . Estimate relative position from X-rays provided Show X-ray to patient for better explanation

Mr Smith, as you can see on this X-ray, this discoloured tooth is a milk tooth which did not fall out and it has prevented your permanent tooth from coming out into its normal position and now the permanent tooth is stuck inside the bone. About 2% of all people have such a problem and most often this happens with an upper canine tooth on one side like in your case. This tooth is fully formed and cannot move from its position on its own so it will not come out if left alone. Explain treatment options

However, there are some options available to you on how we can solve this problem: The first option is to modify the milk tooth adding tooth coloured filing material to it for it to look more natural. The advantage is that it is a relatively simple treatment. However, the white filling may stain and need repairing from time to time. We will have to keep an eye on this area for problems like damage to the root of the nearby tooth or maybe swelling. If one of these happens, we’ll have to remove the underlying tooth with a surgery. In case the milk tooth has some problems we cannot resolve, we could remove the milk tooth and take support from a nearby tooth for attaching an artificial tooth which replaces the missing one. There are various types of bridges and I can explain them to you in more detail later. In general, bridges provide a good aesthetics and are fixed. However, drilling a healthy tooth may be needed which present a risk of this tooth having some complications and needing further treatment in the future like a root canal or other types of crown. Explain the advantages to refer you to an orthodontist

Another option is to refer you to a specialist (orthodontist) who could assess this problem in more detail. Initially, he may need to take some more X-rays from different angles to locate the exact position of this tooth. He will then suggest some more treatment options to you such as: Remove the milk tooth and expose the underlying tooth with a surgery for it to be slowly pulled with braces placed inside your mouth. This works best if the tooth is not greatly displaced and you must keep your mouth very clean when you are wearing braces to avoid decay. The specialist might also decide to remove the tooth below, especially when it is causing some problems, and we might keep the milk tooth, modify it or remove it and replace it with a bridge. Sometimes, the specialist might even take out the tooth below and plant it in the place of the milk tooth, this is a very complex treatment and I don’t have much knowledge on it. Thank you. Do you have any questions?

26

12. Discoloured upper right central incisor Patient has a history of trauma during sports. Had an RCT and tooth has been filled with amalgam as post-obturation filling. Give a treatment plan. Use patient’s name, communicate empathically, and avoid jargon

Good morning Mr Smith, I’m _______, one of the dentists here today. How can I help you? I understand that you are here for the problem of your discoloured tooth. Is that right? I understand that you’re worried about your appearance and can explain to you about possible treatment options. Take the past history of the complaint if asked to do so

o Since when has this tooth been discoloured? o When was the silver filling done and why? o What was treatment done before the filling? o Have you had any pain in this tooth after the treatment? If the tooth is dark because of blood pigments or root treatment, explain the reason for discolouration like this: o This discolouration can be due to those dead tissues or blood break down products that passed to the next layer of tooth called dentine and are showing through. It can also be due to the discoloration of the filling material used. Explain treatment options with advantages, disadvantages and risks

As your tooth may look dark because of tis silver filling, we will first remove the old filling. If you are happy with the appearance, we can give a white or tooth coloured filling called composite. Advantage is that it will improve the appearance and is cost effective compared to other options. Disadvantage is that it can discolour later. There is a risk that the material shrinks or breaks. If you are not happy, we could try a tooth whitening treatment (or bleaching) in which we apply some chemicals (bleaching agents) on the tooth to whiten it. We can either apply them in the surgery in a few appointments, or we can fill up the space for the filling with these chemicals and seal it. This will be done once a week on 2-3 appointments. We call this walking bleach and when the tooth becomes whiter, we can give a white filling. Advantage is the improvement of aesthetics and there is no tooth preparation involved. However, there are chances of the tooth becoming darker again later and is a disadvantage. These chemicals are caustic, so we’ll have to be very careful in applying.  There is a risk of destruction of roots of the tooth because of the chemicals used. (There is also a chance of sensitivity in the teeth- said this if the tooth is not RCT treated) If the appearance is still not satisfactory, another option is to go for something called veneers, which is like a facing, cemented on your tooth. For doing this, we’ll first remove the front part of the tooth in  a specific shape and paste a thin shell made of porcelain or composite. The advantage is excellent aesthetics. Disadvantage is that it requires minor tooth shaping and includes a risk of chipping off in few cases. If the tooth has a large filling and does not have much strength to support a veneer, we can cut the tooth a little more all around and give you a cap or crown which covers the entire surface of the tooth. The advantages are: it will give strength to the remaining tooth and excellent aesthetics. But a lot of tooth removal is involved when shaping it and it is quite expensive. As it is root treated, the tooth would be brittle and there is a chance of tooth breakage during the tooth preparation. So, Mr Smith, these are your options and now it’s your choice. Before we proceed to any treatment, I may have to take an x-ray to assess the quality of the root filling. Do you have any questions? Thank you.

27

Mr John Brown is a 35 year-old medically fit patient. He is employed as a school teacher. Mr Brown originally presented with a painful and discoloured UL1 5 years ago, following a road traffic accident. You had successfully performed root canal treatment for the tooth. Mr Brown has noted a gradual deterioration in the colour of the tooth, and this has led to him attending today. Explain to Mr Brown why the UL1 has discoloured. Discuss the various treatment options available to manage his aesthetic concerns. Please outline the relevant advantages and disadvantages for each option. Patient instruction 5 Years ago, you were in an accident and nothing was done about your tooth. A year ago, you noticed some pain and went to the dentist who advised you that the nerve was dead in the tooth and suggested root canal treatment. This was done and the tooth went a little grey which you were not too bothered about out. However, now the colour had darkened and you would like to go through some possible options to remedy this.

28

13. Missing tooth Patient has missing upper front tooth, using removable dentures and wants a fixed option Patient lost central incisor and is. He/she is not happy about wearing removable dentures and needs to know fixed options. The patient smokes 6 or many cigarettes a day. Use patient’s name, communicate empathically, and avoid jargon

Good morning Miss Fiona, I’m ______, one of the dentists here today. I gather that you are here to  know the replacement alternatives for your missing front tooth, other that removable options. Is that right? Take the past history of the complaint if asked to do so

When and how did you loose this tooth? Since when are you using the removable dentures? What is your main concern today or what do you dislike about the existing dentures? Have you consulted a dentist for this problem before? Is there any other missing tooth nearby? Patient gives h/o Trauma, restored with post crown, got fractured and now using an RPD Take a brief medical history (esp. Diabetes) and social history (esp. Occupation, smoking) Explain treatment options with advantages, disadvantages and risks

I understand your concerns and I will try my best to help you. There are several treatment options available to you such as adhesive bridge, three-unit bridge and implants. I will explain each one with pros and cons, so that you can make your mind up. The first one is an adhesive bridge, where we slightly modify the back of a tooth next to this space and stick a metal extension which is like a wing from the artificial tooth, with a special adhesive. Advantages are it is simple and cheaper, we’ll not have to cut much of the nearby tooth, we also don’t have to numb any tooth. The disadvantage is that the life of this bridge is about 5 years which is lower compare to other fixed options and sometimes metal may show through between the teeth. There is also a risk of it debonding in few cases. The next option is of a conventional fixed-cantilever or fixed-fixed bridge which consists of an artificial tooth taking support from both the adjacent teeth. For this we numb the nearby teeth and cut either one or both of them in a specific shape and we then stick caps that cover them for supporting this bridge. The advantages are that this lasts longer than the adhesive bridge (8-10 years), don’t come out easily and the appearance is better. The disadvantage is that they are expensive and invasive as it requires removal of healthy tooth structure. When preparing these teeth, there is a risk that the inner core called pulp (that contains blood vessels and nerves) might be damaged in a few cases due to the preparations and may necessitate complex treatments like root canal treatment which I can explain later. The risk of pulp damage is 1 in 5 or 20%. The next fixed option is a single tooth implant which would be done by a specialist. A small metal rod is placed directly in the bone by surgery and an artificial tooth is attached to it after 3-4 months. The advantages are they are excellent in appearance, usually very successful long term permanent option and there is no need to drill the nearby teeth. But they are very expensive, need good bone support, involve surgery and are done in steps and for about 3-4 months they need temporary replacement with removable dentures. As implant fixation involves surgery, it has risks of all post-operative complications like pain, swelling or may be damage to nearby tooth during surgery. The success of implant goes down in your case because of smoking. So if you opt to go for this option, you may have to stop smoking to increase the success of treatment. Plastic or metal partial dentures are made quickly, simple and cheap but need to be removed daily and cause loss of bone below them and need to be readjusted regularly. So those are your options and we can give any, depending on your choice. Do you have any questions? Thank you.

29

30

14. Erosion - causes and management Use patient’s name, communicate empathically, and avoid jargon

Good morning Mr Jones, I’m _____, one of the dentists here today. How can I help you today?  (Pt. complains of worn and short teeth) Take the past history of the complaint

Which teeth are worn out, the teeth in front or in the back? How badly do you think they are worn? Are you worried about appearance or pain and function? When did you first noticed it? Has it increased since then? Is there any sensitivity to hot or cold or any pain? Have you been to a dentist for this problem before? Do you have any health concerns? Are you taking any medication? Do you have tummy upset? Do you consume a lot of fizzy drinks, fruit juices or wine? Do you have any habits of sipping, swishing, frothing or holding drinks in the mouth? Do you have a habit of closing your jaws tightly or grinding the teeth? Do you think you brush your teeth too hard? Do you consume alcohol? How many drinks a week? (Determine the cause based on the history - dietary, parafunctional habits, abrasion or medical problems - gastric reflux or bulimia) Explain condition and cause

Mr Jones, slow wearing of teeth with age is a normal process. But in your case this wearing has been much faster. This can be caused by a combination of many factors which include excessive rubbing of teeth against each other, toothbrush with excessive force and, more importantly, acids which dissolve the hard part of the teeth. These acids might come from within the body as acid reflux from the stomach or from fizzy drinks or acidic fruit juices. So we will first need to understand which of these are the main causes in your case. Monitor tooth surface loss

We’ll need to make some copies of your jaws to make study models, clinical records, and some pictures regularly to check how fast are your teeth rubbing off. Dietary analysis

We’ll give you a sheet on which you’ve to note down all the items you eat or drink for three to four days, including the weekend. Treatment of underlying cause, treat sensitivity and discuss prevention

I will have to refer you to your GP to check if there is any problem related to tummy upset. You’ll also have to do some modifications to your diet (Dietary counselling) – Acidic foods and drinks should be taken only at mealtimes; reduce frequency and intake of acidic foods and drinks; finish meals with something to neutralise acid, like cheese or milk; as salivary flow is almost zero at night, avoid acidic food and drink especially before going to bed. Avoid prolonged sipping and holding fizzy drinks prior to swallowing. Do not brush soon after fizzy drinks or after vomiting as it washes away the tooth surface sooner. So wait at least for 30mts. As far as the sensitivity is concerned, we’ll apply a gel called fluoride varnish on your teeth regularly and prescribe you anti-sensitive toothpastes. We’ll give you a soft bite guard which you’ve to wear in the night and we’ll give you some powder to  put in it when you wear (small amount of Sodium Bicarbonate Powder or Milk of Magnesia must be placed in the guard, to neutralise any refluxed acid pooling in it) If the problem progresses or you are very concerned with the appearance or you feel the sensitivity is unbearable, we can give tooth coloured fillings or crowns or if they are excessively worn we can modify them and take their support for dentures. Refer to restorative specialist for more complex treatment We’ll have to regularly monitor the condition of your teeth which is very important to see if the problem is increasing. Do you have any questions?Thank you.

31

15. Angina pectoris – take history and give diagnosis A 75 years old patient has to undergo extensive dental treatment. He suffers from chest pain. Take history and give diagnosis (h/o of angina, has GTN spray and tablets, does not use them, now gets up in the middle of the night breathless; so diagnosis is unstable angina pectoris) Use patient’s name, communicate empathically, and avoid jargon

Good Morning Mr White. My name is ______. I’ve heard that you have chest pain. I would like to ask  you a few questions to know more about it, is it all right with you? Take the past history of the complaint

History of presenting illness: Site: where exactly in your chest you feel the pain? Can you point it to me? Onset: when did it start? Characteristics: can you describe the pain to me? What exactly do you feel when you have this pain What sort of pain is it? How strong is this pain? How often do you feel the pain? Is that becoming more frequent, or more painful than when it first started? Radiation: Does the pain spread to other parts of the body such as your throat or arm, jaw, stomach? Does it hurt somewhere else as well? Associated symptoms: Do you feel anything else when you have the pain, such as breathlessness, choking feeling, sweating or feeling sick? Do you have a burning feeling or pain on breathing? Do you get a funny taste in your mouth? Timing: How long does it last? (Stable angina pain usually lasts between one and three minutes and should last no longer than 10 minutes, whilst the pain on unstable angina lasts longer, sometimes as long as half an hour). Exacerbating/Relieving factors: What brings it on, what makes it worse, what relieves it? Do you get this pain when you are resting as well? At Night? (they may answer physical exercise (exertion), stress, strong emotion, cold weather, large meals) Brief medical and social history

Medication: Are you taking any medication for this condition Mr White? Which one? Do you have to take it quite often? Do you have it with you all the time? Does the pain resolve when you take it? (this question is important as unstable angina doesn’t  usually go away when medicine is taken) Do you smoke? For how long? How many cigarettes a day? Do you drink alcohol? What do usually you have? How many of those a week? What about your diet? Have you had any injury to your chest? Have you seen your doctor for this problem? Thank you Mr White! Explanation to the patient:

Mr White, accordingly you have told me you seem to be experiencing a heart problem which we call Angina Pectoris (stable or unstable depending on patient’s answer to the history taking). This  happens when blood vessels that feed the heart are a bit clogged and the heart is not getting enough oxygen from the blood for it to function properly. This problem is more common in middleaged man, smokers, having a high cholesterol level, high blood pressure, overweight, diabetics, those who don’t exercise and have family history of heart attacks. You should see your doctor as soon as possible because that is a serious condition and has to be treated immediately (if patient said he hasn’t seen doctor).

32

Your doctor will do a thorough check up and some special tests and then prescribe you some medications. You should also cut down on oily food and red meat and loose some weight and I would suggest you trying to stop smoking. You can contact the NHS Quit Smoking Helpline. As far as dental treatment is concerned, we will be even more careful that you are calm and relaxed and that any treatment that we do is painless as far as possible. Oral anxiolytic treatment may be indicated if angina is precipitated by stress. Patients with ‘unstable’ angina and those with a recent history of hospital admission for ischaemic chest pain have the highest risk, and should not be considered for routine dental treatment in primary care. After the visit to your doctor, please bring any medications or sprays that is prescribed to you for the dental appointments (that’s in case patient has stable angina, if it is unstable he has to be sent  straight to the hospital as he might suffer something more serious at any time). Do you have any questions? Thank you. Left, rather than right, lateral recovery position reduces the pressure on the inferior vena cava (the main vein bringing blood from the lower body to the heart) in patients with increased mass or pressure in the abdomen (e.g. pregnancy). This helps the blood flow back to the heart, to then be pumped around the body again. Pressure on the inferior vena cava, reducing venous return (blood flow to the heart) can result in reduced blood flow, leading to shock.

33

16. Oral Hygiene Instructions Oral hygiene instructions to be given to the patient. Props were kept such as unitufted toothbrush, bottle brush, toothbrush, floss.....but also u should mention about mouthwashes and other things like tooth paste which were not kept. Use patient’s name, communicate empathically, and avoid jargon

Hello Mr Smith, I am _____, your dentist today and I am here to give you few oral hygiene instructions. Is that alright? Explain problems caused by poor oral hygiene

Normally, after a few hours of brushing, a soft whitish deposit is formed on the surfaces of our teeth. We call it plaque and it contains germs combined with food debris and saliva. The germs in this plaque feed on the sugar we eat and release some acids that can damage our teeth causing decay. The germs in the dental plaque can also cause problems like bleeding gums, bad breath, gum disease and eventually loss of teeth. Therefore, it is very important to remove it by effective brushing of your teeth twice a day, once in the morning and last thing at night before you go to bed. Explain toothbrushing

Along with regular brushing, the method of brushing is also important for proper cleaning. You might already be cleaning your teeth regularly but you may need to make some changes to the way you brush. Ideally, you should use a brush with a small head and soft bristles to get into all the areas of the mouth without hurting your gums or scraping your teeth away. Spend around 2 minutes brushing overall and pay attention to cover all areas (inside, outside, and biting areas of each tooth). Give special attention to the areas where teeth meet the gums. You can also use a powered toothbrush which is sometimes more effective at cleaning the teeth. Ideally, we all should use small strokes at 45° at the gum level. However, having a systematic approach to brushing all teeth is far more important and this can be made easier if we do it in front of a mirror. We must brush starting from one side and finishing with the other covering all the surfaces of all your teeth. It is advisable to change your toothbrush every 2 months or when it is visibly worn. It is also very important to a toothpaste containing fluoride as it strengthens our teeth and prevents tooth decay. After brushing, spit out the excess but avoid rinsing as it washes away the fluoride. Explain interdental aids

However, toothbrushing alone cannot reach between the teeth so you also need to use dentalfloss or small brushes called interdental brushes at least once a day after brushing. The floss looks a bit like a cotton thread and you can take a small piece of floss about a foot long. Then wrap it around your index finger or middle finger and keep doing that until you come up with an inch to half an inch long. After that, gently take it and push it all the way below your gum line and curve the floss around to actually look like a C shape, then push it back and forth on your tooth and below the gum and slide it out. Do the same on the other side of your tooth. Your gums may bleed a little when you first begin to floss but this should settle within a few days. The interdental brushes are small tipped brushes which come in different sizes. You can select them according to the space you have in between your teeth. You have to slowly and gently push the brush in the area between your teeth until the other side is reached. Do not apply any force. Initially some bleeding may occur, but this stops over the time. Sugar-free chewing gum after meals is also advisable as it increases the flow of saliva which helps to clear any remaining particles from the meal. You also need to visit your dentists for regular dental checks every 3, 6 or 12 months, depending on your need. Do you have any questions? Thank you.

34

17. Recurrent aphthous mouth ulcers Use patient’s name, communicate empathically, and avoid jargon

Good morning Mrs Jones, I’m ________, one of the dentists here today. How can I help you?  I understand your concern about these ulcers; I know they are usually very painful. May I have few more details about this problem? Take a full history of events leading up to this appointment

Since when do you have these ulcers? Is it the first time you are having this or had similar problem before? How often does it happen? When was the last time you had this problem? Where exactly are these ulcers? Is it a single ulcer or many ulcers together? and how big are they? How long do the ulcers usually last? Has the frequency increased or decreased? How does the affected area look like? (red) Do you have any pain or bleeding from these ulcers? Have you noticed something in particular which brings on the ulcers or causes them to increase such as eating some foods or taking specific medication or when you are stressed or menstruation? Take pain history

onset, character, intensity, site, duration, radiation, aggravating and relieving factors) Have you been to the doctor or taken any medicines for this problem? If yes, did the doctor ask for any tests e.g. blood tests? Take a brief medical history

Are you taking any medicines for any other problem? Do you suspect of pregnancy or are you taking any contraceptive pills? Have you recently changed pills to different brand or different pills? Do you suffer from anaemia? Any recent blood loss or any problems with your menstrual cycle? Do you have any major health concerns, allergies, tummy upset or any skin problems? Is anyone in your family having a similar problem? Do you smoke? If not, have you recently quit smoking? (Praise for quitting smoking – It is important for your health). Have you been stressed due to any reason lately? Management

Based on what we discussed, the problem you are having is likely to be common mouth ulcer which we call (recurrent) Aphthous Stomatitis. It affects about 25% of the population sometime in their lives. There are different factors which can cause these such as stress, genetic factors or vitamin deficiencies. (If traumatic ulcer, say this is because you accidently bit or due to this sharp tooth). The ulcer will heal on its own, but to relieve your discomfort, I can prescribe you carmellose gelatin paste. You can apply this 3-4 times a day on the ulcers (If single ulcers present). If the ulcers are present in multiple areas, I can prescribe you with benzydamine hydrochloride mouthwash. Take 15 ml, rinse it around and spit out. You can also use chlorhexidine mouthwash to prevent secondary infections. In addition to this, some simple measures like avoid spicy foods, acidic fruit juices, crisps and carbonated drinks. Consider drinking with a straw when ulcers are present. I may also need to refer you to your GP for a blood test as these ulcers can be due to some blood deficiency. In your case it is probably due to (explain the reason)…or refer to GP for stress management. If patient coming with ulcers more than 3 weeks then say that I may refer you to a specialist called oral and maxillofacial surgeon who may do some special tests by taking a sample of these and examining under microscope, which is called biopsy. Okay I will see you then after two weeks to check on their healing and we will take it from there. Do you have any questions? Thank you.

35

Candidate instruction You are asked to take a history and formulate a management plan for a patient who has come to see you regarding recurrent aphthous ulceration in a 24 year old female called Miss Watts. Patient brief You are Miss Watts, a twenty four year old female who has been suffering from small ulcers in the mouth for the last year. The ulcers are less than tern in size and never last more than 10 days and are associated with your menstrual cycle. You have particularly heavy periods and changed your contraceptive pill at about the same time that the ulceration started. Do not offer up the information about the contraceptive pill or your heavy periods unless you are asked. You are a non-smoker, you are not under any stress, no one else in your family has this problem and you take no other medication

36

18. Irreversible pulpitis Use patient’s name, communicate empathically, and avoid jargon

Good morning Mr Jones, my name is_____, I’m one of the dentists here today. How may I help you?  I understand your discomfort. May I ask you a few details of this problem? Take pain history - SOCRATES

Where do you feel the pain? Can you point to the tooth where you have this pain? Since how long have you had this problem, and how did it start? What brings it on? Is there any effect when taking hot or cold foods? Do you have pain on biting? Can you please describe what type of pain is it, sharp or dull? Is the pain continuous or does it keep coming and going? How long does it last? Does the pain spread anywhere else? Is there anything which increases or reduces the pain? Has this pain changed in the last few months? Does the pain increase on lying down? Does it disturb your sleep? Did you take any medication like pain killers or sought any treatment for this pain? (Full history of events leading up to this appointment) Are you taking any medicines, for any other problem or have any major health concerns. (Brief medical history) Diagnose irreversible pulpitis and provide basic explanation of disease and aetiology

According to what you’ve told me, the problem you are having seems to be that this tooth has been decayed for some time and the inner core of your tooth, called pulp is infected and dying. This core or canal contains blood vessels and nerves and that is why you are having this pain. At this stage it is called irreversible pulpitis because it is beyond repair. Provide treatment options, advantages, disadvantages and risks

There are two ways to solve the problem you’re having Mr Jones: The first option is to remove this dying tissue, clean the canal inside your tooth with special instruments and seal the empty space with a rubbery material called gutta percha. This is called “root canal treatment” as the procedure involves treating the canal inside your tooth for you to be  able to keep it in your mouth. The advantages of this treatment are that the tooth can be saved, pain is relieved and the success rate is around 85 to 90%. The disadvantages are: it is expensive and multiple visits are required. During this treatment there is risk of small instruments getting broken inside the tooth or sometimes they perforate then tooth. In such cases the treatment may result in failure. The second option is to remove the tooth. advantage is that you get rid of pain quickly and in a single visit. The disadvantage is that you lose your natural tooth and also need to go for replacements of that if you wish. That will bring additional treatment costs. There is a risk of post extraction pain, swelling or infection. At the minute, pain killers will only work for a few hours and the only way to treat this pain is to remove the cause. So to get rid of this problem, we’ll have to remove the infected inner tissue from  the tooth as soon as possible. However, before we start, I may need to take an x-ray to understand extend of decay. Alternatively we can also numb the area and take the tooth out. If we do not treat the tooth, the pain will not be relieved and there are chances that pus might form below the tooth causing a very painful swelling and even fever and bigger complications over time. Do you have any questions? If you’ve completely understood the whole procedure, I would like you to choose a treatment. Thank you.

37

19. Lip laceration by a colleague 1. A colleague comes to you saying he has lacerated the patient’s lip 5 minutes ago. How would you  proceed? 2. The hygienist injures her patient’s lip and has come to you for advice. How would you proceed? Reassure and calm colleague down

Calm down and tell me what happened? Ok, this was an accident, and such accidents can happen to anyone in our profession regardless of how careful we are and it is very unfortunate it has happened with you, please don’t panic. Is the patient still in the surgery? Is he alone? How did it happen, when did it happen? How deep is the wound? Did you inform the patient? Advise what to do

I understand you are distressed; however, it is not right to run away from your professional duties towards the patient, it is negligent and unethical. You should go back to the patient, apologize for this mishap and explain that it happened by accident; explain that it could be due to poor visibility or poor access, and you are going to take care of him. Check the patient’s medical history to see if he has any bleeding problems and give him the necessary emergency treatment. Try to stop the bleeding by applying pressure with gauze, give sutures if needed. If any redness or swelling apply cold pack. Reassure the patient If you cannot control the bleeding, take him to the Accident and Emergency of the hospital immediately. Complaint procedure and recording of the incident

If the patient is unhappy and wants to make a formal complaint, explain the complaints procedure. Inform your practice manager about the accident and the steps you have taken. Note down the details of the accident and the steps you have taken in the Accident Book/ the RIDDOR sheet and the patient’s notes. Ask your nurse to sign it and ask the patient to countersign it. Also write the referral note if you need to take him to the hospital. Call up your indemnity (DDU or Dental protection) and take their advice on this matter. At this time, the best thing to do is to treat the patient yourself because it is your legal and ethical obligation to treat this patient. Also, if any problem comes up later, everyone will understand that it happened by accident and you did the right thing and treated the patient. If you still feel you are not able to treat the patient, you’ll have to take the patient’s consent and if he  allows, only then I can intervene. After you have treated the laceration, arrange a follow up appointment. If sutures are placed, they may need to be removed after one week. Is there anything else you want to know? Thank you. RIDDOR - Reporting of Injuries, Diseases and Dangerous Occurrences Regulations

38

20. Dietary advice and diet analysis Diet sheet or diet analysis chart

Ideally recorded over 4 days including two working days and two leisure days or weekend. It should contain time, content and amount of all foods and drinks consumed as well as toothbrushing times. 1. Ring the main meals 2. Underline all sugar intakes as well as acidic drinks in red. 3. Identify in-between meal snacks and note any associations. Dietary advice to the patient – for caries

After looking at your diet sheet, I’ve noticed that you are taking a lot of added sugar every day in tea,  candies, drinking a lot of fizzy drinks etc. The germs present in the mouth react with sugars and cause tooth decay. Every time you eat or drink anything sugary, the chances of decay increase. To improve the health of your teeth, you need to make some changes to your diet: o Reduce amount and frequency of sugar intake. o Avoid in-between meal sugary snacks. o Restrict sweetened drinks to mealtime and not in between meals. o Eat sugars with the meals instead of dividing them through the day. o You can limit sweets to once in the week. o Don’t finish you meal with sugars. But if you eat a sugary meal eat cheese or chew sugar-free chewing gums afterwards (xylitol) as they increase saliva and help to clear the sugars faster. o Avoid sugary foods and drinks near bedtime (1 hour) as saliva production is lower asleep. Theory questions - studies

Which studies prove the relationship between caries and sugar? (For explanation to the mother) Vipeholm Study and Turku sugar study. The Vipeholm study was done to investigate the association between sugar consumption and dental caries. It concluded that the consumption of sugary foods is associated with a higher caries incidence. Increasing the frequency of sugar intake also increases caries incidence. Lowering sugar intake reduces caries incidence. Consumption of sticky sugar-containing between meal products can be associated with high caries risk. Several dietary factors are associated with caries incidence: Amount of fermentable carbohydrate eaten Frequency of eating meals and snacks Sugar concentration of food Length of interval between eating Physical form of carbohydrate Sequence of food consumption Oral retentiveness Turku Study- A longitudinal study to evaluate the caries incidence as affected by partial substitution of dietary sucrose (S) with xylitol (X), the effects of S- or X-containing chewing gums were compared during one year. The results show a profound difference in caries rate between the two groups. The findings clearly indicate a therapeutic, caries inhibitory effect of xylitol. Name 4 forms of carbohydrates that are harmful - Sucrose, Fructose, Glucose, Maltose Names of 4 commonest substances to look for and avoid on labels - No added sugars – It means there are sugars already present even if none has been added. Check for content of sucrose, fructose, glucose, maltose, and other names of sugar such as hydrolysed starch, invert sugars, corn syrup, honey, raw cane sugar Dietary advice to the patient – for erosion

After looking at your diet sheet, I’ve noticed that you are consuming a lot of acidic drinks every day as fruit juices, wine and fizzy drinks etc. Acidic foods/drinks/fizzy drinks can harm the teeth, they dissolve and weaken the hard part of the tooth and remove the surface of the teeth making them look bad and sensitive. To improve the health of your teeth, you will have to make some changes to your diet: Reduce intake of acidic foods and drinks. Restrict acidic drinks to mealtime and avoid consuming such foods and drinks in-between meal. Avoid acidic foods and drinks close to bedtime (within 1 hour) (Artificially sweetened diet drinks are as acidic as normal fizzy drinks so avoid them as well.) Avoid frothing and swishing drinks around the mouth. If you have to take fizzy drinks, drink them quickly or use a wide bore straw placed at back of the mouth. Do not brush soon after taking acidic foods or drinks. Finish meal with neutral foods like cheese or milk which reduce action of acids. Sugar free gums (xylitol) and cheese increase saliva and help to reduce destruction of tooth by washing away the sugary and acidic foods and drinks Do you have any questions? Thank you.

39

Questions: Diet chart was provided and you were asked what 5 things we can modify in the chart. The diet chart was of a 9 year old child with a high cariogenic diet. The mother of the child wanted evidenced for the relation between caries and sugars so we had to write about studies that prove the relationship between sugar and caries. Also give names of 4 commonest substances that the mother should look for and avoid on labels. Erosion case - Explain the cause and management. A diet chart was provided. Diet chart for erosion - fruit juices, coke, grapefruit juice, red wine, vinegar salad dressing....explain the patient and advice against erosion...the patient had a habit of swishing and frothing the drinks. Also ask about eating disorders. The main cause of tooth decay is not the amount of sugar or acid in the diet, but how often it is eaten or drunk. The more often your child has sugary or acidic foods or drinks, the more likely they are to have decay. It is therefore important to keep sugary and acidic foods to mealtimes only. If you want to give your child a snack, try to stick to cheese, vegetables and fruit. Try to limit dried fruit as it is high in sugar and can stick to the teeth. It is also worth remembering that some processed baby foods contain quite a lot of sugar. Try checking the list of ingredients: the higher up the list sugar is, the more there is in the product. Sometimes, these are shown as fructose, glucose, lactose or sucrose. Thorough brushing for two minutes, twice a day, particularly last thing at night, will help to prevent tooth decay.

40

41

42

21. Bottle caries A 3-year old patient presents to you for the first time with his mother. The child has never seen a dentist before and has no prior experience of having dental treatment. You need to discuss preventive advice with the parent and management options. Patient instructions: You are the parent of a 3-year old child who has decay as shown in the photograph below. You are going to see a dentist for the first time to seek treatment. The candidate is expected to discuss the cause of the decay with you without the use of technical terms and discuss ways in which this problem could be prevented in the future as well as treatment options. They should also show empathy.

Marking sheet: Topics to be covered

Done

Not done

1

Introduces themselves to parent

1

0

2

Asks what concerns are

1

0

3

Able to recognise nursing caries and explain to patient

1

0

4

Discusses causes i.e. bottle of milk or comforter at night, lack of saliva production at night therefore increasing risk of decay

1

0

5

Avoid use of jargon

1

0

1

0

1

0

1

0

1

0

1

0

6 7 8

Advise stop bottle of milk at night. Discusses alternatives i.e. water in the bottle instead and gradual weaning off bottle to cup Discusses OHI i.e. brushing twice daily, use high concentration fluoride toothpaste and supervised brushing Shows empathy throughout

Discussed treatment options i.e. no treatment, restoration of teeth or extractions Discusses management options such as treatment under local 10 anaesthesia, relative analgesia (N2O) or general anaesthesia. 9

Total Notes for examiner: Make sure candidates show empathy and explain terms in a friendly way. Mina is happy to add more fluoride dosages for 3-year old or any other preventive treatment if needed be.

43

22. Lichen planus/lichenoid reaction Picture of patient was kept. Patient was on ibuprofen (NSAIDS) for a long time against pain from broken hand. Patient is a non-smoker. Explain to patient about the lesion and advise accordingly. Use patient’s name, communicate empathically, and avoid jargon

Good morning Mrs Patel, my name is____, I’m one of the  dentists here today. How may I help you? Take the history of the complaint

When did you notice it? For how long have you had this problem? Is it painful? Bleeding discharge or itching? Does anything make it worse of better? Have you had any dental restoration put near where the lesion is recently? Are you on any medication, either prescribed or over the counter? Do you have lesions in other parts of your body? Any family history? Do you smoke? If yes, how many a day and for how long? If not, have you ever smoked? Are you under any stress? Information

Based on what we discussed and the presentation of the lesion you have, the problem you are having is likely to be a common condition called lichen planus. Some people have it on their skin, some in their mouth and others on their skin and in their mouth. It can last for many years and it is not infectious. The cause is not known, but it can be set off by some drugs such as antimalarials, antidiabetic drugs, NSAIDs) or dental restorative materials (amalgam and gold) Management

A biopsy, which involves taking a small sample of tissue, can give a definite diagnosis and exclude other causes. Blood tests can help exclude other conditions. If there are amalgam restorations adjacent to the lesions it may be advisable to have them replaced. If drugs are likely to be implicated, then it may be advisable to liaise with the patient's general medical practitioner to try alternatives. Regular monitoring is required to ensure that the disease does not progress. A subtype that affects the gingiva (gums) may make toothbrushing difficult. Spicy or salty food may make the lesions more sore. Treatment (is aimed at alleviating the symptoms and may involve)

Avoid spicy and hard foods Benzydamine hydrochloride mouthwash (Difflam) to numb the sore area. Topical steroids, eg betamethasone mouthwash, hydrocortisone (Corlan pellets), triamcinolone acetonide. If these do not provide relief then specialist care may be needed, which could involve steroids injected into the lesions themselves. In severe cases, steroids It can take between three months to one year for it to disappear. However, although the likelihood is small, we will monitor the lesion as it has the potential to turn malignant.

44

Case 2 This is Jenny Brown, a 32 year old legal secretary. She is not known to your practice and has come in today complaining of a sore patch on the right side of her mouth, which has become increasingly uncomfortable over the past week. She is obviously very distressed and asks you to have a look. On examination, you see an erythematous area with irregular white striae on the buccal mucosa opposite the lower right first molar. There are no ulcers. The LR6 has a large occluso-buccal amalgam. The occlusion is sound and the remainder of the oral cavity is completely normal. Assuming you have performed an adequate examination, complete your history and explain to the patient what you think the likely diagnosis is, and your outline your management. Patient instructions You are a 32 year old legal secretary who comes in today with a red-white sore area on the cheek opposite your first right molar. It is causing a burning pain which has got worse over the past week, although it may have started a fortnight ago. It is made worse by spicy foods and nothing helps to ease it. You had a large silver filling put in the tooth next to the cheek about three weeks ago by another dentist and you think symptoms started around then. You have no tooth ache, and you have not bitten your cheek or felt the filling as sharp. You have no medical problems, have never taken any medication and do' not smoke. You rarely drink. You are concerned that this may be cancer and are very anxious. The dentist has already performed an examination. The dentist should complete their history and address your concerns, as well as outlining his/her management plan to you.

45

23. Avulsed tooth – phone call Introduce yourself, communicate empathically, and avoid jargon

Good morning, ______ speaking, the dentist for today. How may I help you? I’m sorry to hear that. How is your son now? Take a full history of events and a brief medical history

Before that I would like to ask a few questions to be able to guide you properly ok? How old is your child? Is there any other injury? Is he bleeding? When did it happen? Was there any loss of consciousness, nausea or vomiting? Does he have any medical problems such as heart or bleeding problems or allergies? Is he under any medications? Is he up to date with his tetanus immunisation? Where is the tooth now? Is it intact or broken? If broken, have you got the pieces? Is there any chance your child may have swollen it? Is it on your hands or stored in a medium? If not, would you be able to put it in a glass of milk? Explain how best to proceed

As the tooth is not broken, the best thing to do now is to place the tooth back into the socket where it was originally. The sooner this is done the better is the outcome for the tooth. If you are happy to do this and your son accepts it, I can give you all the instructions and guidance over the phone. If you are not sure you can do it, is there anyone around you who can help? Guidance

Hold the tooth by the white shinny part that normally shows in the mouth, not the root which is the elongated part of the tooth. If the tooth is dirty, quickly rinse it with milk or tap water. Do not wipe it off or use soap and don’t let it dry out. If the child’s mouth is dirty, please ask them to rinse their mouth with water and spit it out. Holding the tooth by the crown, with the smooth convex surface facing you, replace the tooth in the socket where it came from. Ask the child to gently bite on a piece of gauze or clean handkerchief to seat the tooth correctly to the level of the adjacent teeth. Are you able to do it? If yes say. Good! Now, come to the practice to see me as soon as possible so I can continue the treatment. Here I will assess the wound, immobilise the tooth and maybe take an x-ray. If patient is reluctant to do it and prefers to come to see you advise to put the tooth in milk and come to the dental surgery as soon as possible. Thank you, see you soon, bye.

46

You receive a phone call from a mother/father of a 10 year old boy. The parent tells you that their son has been playing rugby at school and collided with another player. As a result he has knocked his upper right central incisor (UR1). The parent is very distressed and would like your advice as to what they should do. Actor instructions You are an anxious mother/father of a 10 year old boy. Your son has been in an accident at school whilst playing rugby and his upper tooth has been knocked out. You are distressed and need advice. Examiner instructions: Candidate needs to demonstrate ability to be calm under pressure and give clear and concise advice to the parent.

47

24. Needle-stick injury First aid

Encourage the wound to bleed. Wash the area with water and soap, but do not scrub the wound. Cover it with a waterproof dressing. Stop the treatment. Seek urgent medical advice (for example from your Occupational Health Service), as effective prophylaxis (medicines to help fight infection) are available. Report the injury to your employer. Fill in a RIDDOR sheet and submit to health and safety. RIDDOR - Reporting of Injuries, Diseases and Dangerous Occurrences Regulations (1995) Assessment of the patient

If possible, another member of staff should assess the viral carrier status of the source patient, and hence the likely risk of transmission of an infectious disease. Baseline bloods may be required from the source patient for storage and possible future testing. This will have to be done by another healthcare professional to avoid conflict of interest, and only after proper consent has been obtained from the patient. If the source patient is known to be HIV positive, or hepatitis C positive, immediate specialist advice must be sought and antiretroviral drugs taken prophylactically as soon as possible. What to do?

If there is a worry that the source patient may be high risk for an infectious disease, then urgent advice should be sought according to the local rules. Each primary care trust will have at least one designated specialist (often the consultant medical microbiologist) who can be contacted for advice on post-exposure prophylaxis (PEP). Details of how to contact them should be clearly displayed in the dental practice. Local arrangements should be in place at the practice to enable follow-up and prompt action to be carried out. This may involve going to the nearest accident and emergency department, where you must have the following checked: Tetanus status - If inadequate, a tetanus booster will be needed. Hepatitis B status - If previously immunised, antibody titres should be checked. If low, a booster vaccine is needed; if very low, then immunoglobulin will be needed and a vaccine course should be started. If not previously immunised (this should not be the case for healthcare workers in the UK), hepatitis B immunoglobulin should be given and an active immunisation course started (first vaccine in accident and emergency and arrangements made for subsequent doses). Counselling and follow-up should be arranged as necessary. Follow-up

Ensure there is adequate follow-up of both care worker and donor. The care worker in particular will require early involvement by the Occupational Health service. They may need specific advice about having to take sick leave if medication is required and the possible requirement for psychological support. Record the incident

Fill out accident book and complete critical event audit. How can subsequent events be prevented? The incident should be recorded in the practice accident book. Details recorded should include: Who was injured? How has the accident occurred? What action was taken? Who was informed and when? Who was the source patient? Further information

Risk of acquiring hepatitis B following a needlestick injury from a carrier has been estimated at 2-40%; the risk of hepatitis C is believed to be 3-10%. The risk of acquiring HIV after a needlestick injury from an HIV-positive source is 0.2-0.5% but may be higher if significant volumes have been injected.

48

The patient in front of you has attended for a restoration on the upper left first molar tooth. Having administered a buccal infiltration, whilst attempting to re-sheath the local anaesthesia needle, you suffer a needle-stick injury. Act out in front of the patient how you would manage this situation, and describe to the examiner (where necessary) the sequence of events and actions which you would follow in managing this accident, in both the short and long term.

49

25. TMJPDS Use patient’s name, communicate empathically, and avoid jargon

Good morning Mr Jones, my name is_____, I’m one of the dentists here today. How may I help you?  I understand your discomfort. May I ask you a few details of this problem? Take pain history - SOCRATES

Site - Where exactly is the pain? Can you point it to me? Onset - When did it start? How often do you feel the pain? Characteristics - Can you describe it to me, what exactly do you feel when you have this pain, what sort of pain is it? Is that becoming more frequent, or more painful than when it first started? Radiation - Does the pain spread to other parts of the body such as your throat or arm, jaw, stomach; does it hurt somewhere else as well? Associated symptoms – Is there any noise or crepitus from the joints, does your jaw get stuck when opening your mouth? Is there any swelling, fever or discomfort? Timing: How long does it last? When does it come usually? Exacerbating/Relieving factors - What brings it on, what makes it worse, what relieves it? Do you get this pain when you are resting as well? Severity: on a scale of 1-10 how severe is the pain? Have you ever sought treatment for this problem? Brief medical and social history

Are you medically fit and well? Are you taking any medication, either prescribed or over the counter? Are you currently seeing your GP for any medical condition or treatment? Have you seen your doctor or dentist for this problem? Do you smoke or have smoked in the past and is trying to quit at the minute? What about your diet? Are you under any form of stress or hardship at the minute? Do you grind or clench your teeth? Explanation to the patient and Management

From the information I gathered, it is likely that you may be suffering from TMJPDS. This is a very common condition which affects your jaw joint and muscles in one or both sides. It is usually a benign muscular condition so it is not harmful and does not cause any long term damage. Symptoms come and go, often worse when you are anxious. TMJPDS can be presented as clicks, crepitus, pain and restriction of movement, stiffness or locking of your jaw joint, earache, difficulty opening and/or closing your mouth, headaches and difficulty swallowing. Sometimes the muscles also can get a bit swollen and it may also trigger migraines. Most of the discomfort comes from overusing your muscles and joints such as clenching or grinding your teeth. This is usually worse when you are worried, stressed or depressed as well as when grinding your teeth at night. Habits such as chewing pencils, biting your nails, holding things in your mouth and habitual chewing gum can all cause this problem as well. The fact that a white line has appeared on both sides of your cheeks and your tongue has small indentations along the sides are signs that you grind your teeth at night. Grinding alone can be causing the pain and discomfort you are complaining about which may be worse when you wake up or at night. Management of the condition

The condition tends to come and go so it is important to keep the jaws apart and stress levels down to prevent it from happening in the first place. There are many simple and effective treatments for TMJPDS. During the acute phase, when pain is present, it is advisable to follow a soft diet, do gentle jaw exercises, apply heat pads do relief the tension in the muscles and also avoid opening the mouth really wide. If you wish, I can give you analgesics or even muscle relaxants to relieve the pain and tension you are experiencing. To prevent it from keep coming back and also protect your teeth, I would also like to provide you with a bite splint that will keep your teeth apart and allow your jaw muscles to rest. I would like you to return in 4 to 6 weeks for a review appointment. If there has been no improvement or if the condition gets worse, I would like you to come back earlier so we can reassess the situation and possibly refer you to a specialist, who will be able to try more complex treatments. Do you have any questions? Thank you.

50

Patient’s complaint I have been experiencing pain on left hand side of my face. It is centred just in front of my left ear on but occasionally I have noticed it on the right side as well, but not as frequently. History of presenting compliant Question 1 Patient suffering from temporomandibular pain and dysfunction syndrome - take history, give diagnosis and explain the condition. Question 2 (RCS) Candidate instruction This is Michael Brown, a 22 year old chemistry student. He has come to see you because of pain in his face. The pain is associated with area in front of his ear. It is mainly on the left hand side but can occur on the right as well. The pain is worse in the morning, at the end of the day. It also gets painful when he eats some meals and sometimes he will wake because of the pain. He has visited his dentist in the past who have investigated his teeth and have found no dental pathology. He has had a click in his jaw for the last 3 years and occasionally feels his opening is limited but can open mouth fully. Take a concise history and explain to the patient what you think the likely diagnosis is, and your outline your management plan to him. Patient brief Your name is Michael Brown. You are a 22 year old chemistry student. You have been experiencing pain on left hand side of face. It is centred just in front of your ear on left but occasionally you have noticed it on the right side as well, but not as frequently. The pain is worse in the morning and at the end of the day. Eating also exacerbates the pain and occasionally it will wake you up from sleep. The pain began 6 months ago at the time when you were diagnosed with Non-Hodgkin’s Lymphoma. You recognise that you grind your teeth at night. NSAIDS or antibiotics have not helped. The symptoms are not related to hot, cold or sweet foods or drinks. There has been no injury to the area or jaw and there was no previous history of it. There are no other medical issues of medications being taken. You are waking up early in the morning since the Non-Hodgkin’s Lymphoma diagnosis and your a petite is poor and deteriorated. Your mood does vary during the day and you have less energy than normal. You recognise you have been stressed since the diagnosis and treatment for the condition. You also know that your mouth has been dry and that a white line has appeared on your cheeks on both sides, and you have noticed your tongue has small indentations along the side. Your dentist has not found any other causes after investigations on your teeth involving X-rays and applying something cold to each tooth. You also remember something to do with "small electric current". You have had a click for the last 3 years and although you an open your mouth fully, occasionally it feels it gets restricted/limited in how far it opens.

51

52

Skilled OSCEs

53

Intramuscular injection Anaphylactic reaction – IM adrenaline 500µg (0.5mg) of 1:1000 Hypoglycaemia – IM glucagon 1mg How to give intramuscular injection of 2ml of diazepam 10mg solution Examiner will not tell you anything, just do it. Props kept: diazepam vial, syringe, big and short needle, alcohol swab and sponge on which IM to be given, sharp box and gloves Introduction and explanation of procedure - communicate empathically, and avoid jargon

Good Morning Mr John, my name is _________. I understand that you have been prescribed __________and I’m here to give you the medication, is that ok? Before we proceed, I would like to ask you a few questions and explain what I’ll be doing. Are you generally fit and well? Do you have any medical problems known to you? Are you allergic to any drugs, medicines or latex? Are you taking any medicines, prescribed or over the counter? So, about the procedure, I’ll first check the medicine and the right material and get everything ready.  Then, I’ll clean the area on your skin with an alcohol wipe. You will feel a sharp scratch but I’ll let you  know when exactly I’ll be injecting the medicine so you are prepared for it. Are you happy for me to proceed? Okay, thank you. Injection

Say you assume you have washed your hands and then put gloves on. Clean skin with alcohol wipe and wait to dry. Meanwhile… Check name, concentration and dose, expiry date and batch number (Read out both) Break vial (hold it with the blue dot pointing towards you and break it away from you. Pick bigger needle (green - 40mm or 21gauge) and draw medicine, discard needle. Pick another needle (green) or (blue - 25mm 23 gauge), prepare the injection. Stretch the skin to reduce fat layer and facilitate delivery of drug. Warn patient “SHARP SCRATCH COMING NOW” and insert needle at 90° on the rubber thigh pad. Aspirate and then deposit the drug slowly. Remove the needle, place cotton and ask patient to apply gentle pressure for a few minutes. Dispose the syringe with needle in the sharp box (don’t re-sheath). Ask if the patient is ok and say thank you. Viva questions

Where do you give the IM injection? Why? Anterior lateral aspect of thigh (Vastus lateralis), because of easy access to the large muscle and there are no major structures such as nerves arteries that could be damaged, the bulk of the muscle offers easy and rapid absorption of the drug, permits multiple injections. What is the landmark? Hand-span from ilea crest or half way from knee to ilea crest What other sites can you use for the IM injection? Deltoid or gluteus muscles. What is the advantage of the IM? The drug can be absorbed quickly and there is no need to find a vein.

54

Candidate instrucfion Julia Blakeson is a 24-year-old insulin-dependent diabetic. She is known to your practice, and is otherwise fit and well. Her diabetes was only recently diagnosed. She comes to your practice for an extraction, with her BM-stix, and is happy to proceed. As you begin to prepare for the extraction, she tells you she feels faint. She has missed breakfast, mistakenly thinking she needed to be starved from midnight, but has taken her insulin. Very quickly, you notice that she is sweating, agitated and her speech is slow and confused. You check her blood glucose using the finger-prick equipment - her BM is 2.1. As you prepare to get a sugary drink, she stops talking and slumps to the side. What do you think is going on, and how will you manage the patient? To Examiner Must select glucagon, or FAIL Must check ABC, and understand why When drawing up water, assume 5mg/ 5ml- therefore, candidate should only require 1ml in syringe Candidate can 'go through clothing'; no indication for alcohol wipe or swabs.

Current Department of Health (2006) recommendations are that needle length must be sufficient to penetrate the subcutaneous fat layer using at feast 25mm (23 gauge) blue needles or 40mm (21gauge) green needles for adults. Practitioners should hold the syringe like a pen to insert in a dart-like motion, to reduce accidental depression of plunger. The green needle is inserted 2/3 in (leaving approximately 10-13mm out). No needle should be inserted to hub as weakest point. Advise patient to see GP (especially if first hypoglycaemic event) as scope for patient education, review dose of insulin.

55

Give 2ml of solution and give an intramuscular injection to the patient (training arm/pad) 1

Introduction

done not 1

2

Check patient details are correct

1

3

Selection of Instruments: Appropriate syringe Appropriate needles: 1 blue or green for withdrawing and 1 minimum size of blue for injecting. Sterile swabs. Plaster. Current department of health (2006) recommendation are that needle length must be sufficient enough to penetrate the subcutaneous layer using at least 25mm (23 gauge) blue needle or 38mm (21guage) green needles for adult.

2

4

Check the expiry dates of equipment and medications

1

5

Washes hands and uses gloves

1

6

Draws correct amount: 2ml and ensure there are no air bubbles.

1

7

Change needles for drawing up and injecting.

1

8

Clean area with sterile swab " if the skin is ‘clean’ there should  be no need to swab the injection site ‘’If skin is visibly unsoiled and disinfection is still performed, care should  be taken to disinfect the area properly with pre-medicated 70% alcohol swab. the injection site should be cleaned for 30secs with alcohol swab and further allowed to dry for 30secs to ensure that bacteria are rendered inactive and injections are given safely’’.  Guidelines issued by the Infection Control Team, Healthcare Associated Infection & Infection Control Section, Health Protection Scotland (formerly Scottish Centre for Infection and Environmental Health (SCIEH). Issued September 2005

1

9

Injection given at 90 degree to skin. Not jabbing. Practitioners should hold the syringe like a pen to insert in a dart-like motion, to reduce accidental depression of the plunger Use the Z technique or stretches the skin. Z technique involves pulling the skin to one side at the intended site. This moves the cutaneous and subcutaneous tissue by approximately 1-2cm and ensures the injected fluid remains in the intended area.

2

10 Aspirates.

1

11 Slowly injects fluid over 20 seconds. Ideal rate is 1 ml per 10 seconds

1

12 Applies pressure over injection site

1

13 Removes needle using the sharps bin needle remover or alternatively places the syringe and needle in the sharps bin 14 Apply plaster on wound, but check allergy status of patient first

1 1

56

Waste disposal The equipment displayed on the tray has been used on a patient. Please assign the articles to the appropriate tray /bags/ containers in accordance to the infection control guidelines. Six trays present – each marked as: BB - Black bag

WA - Wash and Autoclave

YB - Yellow bag

SD - Surface Disinfection

SB - Sharps Bin

DB - Disinfection bath/Immersion-Impressions

Instruments Scalpel blade Extraction forceps Rubber cup Rubber-dam clamp with thread floss Glove box Impression oral tip Beam aiming device Disposable matrix band Syringe needle X-ray viewer X-ray film packet

Tray

Instruments Scalpel handle (Plastic) Suture needle Rubber-dam sheet with plastic holder Gloves Micro-brush Dappen dish - plastic and glass Eye wear Wooden wedge Half used cartridge Plastic attachment Impression tray with impression (goes in disinfection bath)

Tray

Put each in the correct tray. Yellow Bag Single use items contaminated with blood, saliva or other body fluids (Paper towels, cotton rolls, cling films, used X-ray film packet) Decontamination X-ray viewer Autoclave Handpieces Needles and cannulae Matrix bands Teeth and roots Wooden inter-dental wedges Disposable 3-1 water/air tips Ortho wires and brackets Lab work for disposal Burs and prophylactic cups Compules Special waste Amalgam Teeth filled with amalgam Radiographic developer and fixer Impressions- Rinse with water and then disinfect by inunersion in I% hypochlorite or Perform (Diisocynurate) for I 0 minutes. Re-rinse with water. Spittoon - Clean and disinfect

Black Bin Biodegradable kitchen waste Single use items of personal protection equipment (gloves, masks and disposable aprons) Protective eye wear Metal instruments Dappen dish Sharps bin - anything that can pierce the yellow bin such as Disposable syringes Endodontic instruments Scalpel blades Slow andn fast suction tips L.A. cartridges Drug ampules empty Impression trays Micro brushes and applicators L.A. cartridges (partially discharged) Amalgam capsules Surplus drugs in syringes and ampoules Contaminated plaster models Others Transport to lab in sealed, named bags. Paper and cardboard Packaging material

57

Viva Questions (one mark each) What goes in the yellow bag? Non-sharps clinical waste i.e. anything contaminated with blood/ saliva, but NOT special waste - see below What is the maximum clinical waste allowed for a practice in a year? There is no limit for clinical waste; however, anything over 200kg of Special Waste (X-ray Fixer, lead foil, etc) needs to be registered as a producer of special waste Where should bags of clinical waste be stored before collection? Bags of Clinical waste must be securely stored in a dedicated ventilated and secure area where only designated people can come into contact with it and it is unlikely to contaminate anything else Who collects the waste? Only a Licensed Waste Facility can collect waste, and a Transfer Note must be given for each consignment, to be kept for a minimum of three years

58

Rubber dam Candidate instruction Your patient requires a composite filling on the LR4 MOD. To achieve moisture control you have decided to isolate the appropriate teeth with a winged clamp. It is understood that you are following all infection control and health and safety guidelines including wearing a mask and gloves, this exercise involves rubber dam application including all the steps involved.

59

Hand washing

60

Suture - single interrupted suture Tell examiner that you'll first clean, anesthetise and take consent. Select correct suture material (3-0 silk) and curved reverse cutting needle (triangular cross-section) Pick needle holder, tissue holding forceps and blunt end scissors. Load needle into the holder. (Never touch the needle with hand) Evert edges and place 1st bite of needle at an angulation of 90° to soft tissue. Place second bite on the other side of tear and bring out the needle from the other side and place a knot (2 clockwise -1 anticlockwise -1 clockwise) on the buccal side while squaring the knot. Cut excess suture thread with 0.5mm left with the knot. Dispose needle into sharps box, cut thread pieces into clinical waste.

61

Blood pressure Denture faults CPR BPE Dental charting IOTN classification Referral letter Lab documents Radiograph shown and we have to write like multilocular etc Radiographic faults Treatments for hygienists and dentists Arranging radiographic film Tooth splint Denture candidosis (aetiology,characteristics,treatment and other relevant information) Identifying surgical instruments Cephalometric analysis

62

Plaque Control Record (the O'Leary Index) Disclosing solution is applied to all supragingival tooth surfaces. After the patient has rinsed to remove excess dye, each tooth surface (except occlusal surfaces) is examined for the presence or absence of stained deposits at the dentogingival junction, four surfaces for each tooth. Plaque, if present, is indicated on the appropriate box in a diagram. After all teeth have been scored, the index is calculated by dividing the number of surfaces with plaque by the total number of surfaces scored and then multiplied by 100 to get a percentage of surfaces with plaque present. A reasonable goal for patients is 10% or fewer surfaces with plaque, unless plaque is always present in the same areas. If so, special instructions should be directed toward improving performance in those areas. It is extremely difficult to achieve a perfect score of 0, so patients should be rewarded for approaching it. Some commonly used plaque indices do not require staining the teeth, such as the Plaque Index of Silness and Loe.133 These may seem more convenient to use and possibly more acceptable to patients, but they have some disadvantages for patient education. Identification of plaque is not as quick and easy for the clinician's record making, and because plaque is not stained, it is not highlighted for the patient to see and remove. The Plaque Control Record was developed to give the therapist, hygienist, or dental educator a simple method of recording the presence of the plaque on individual tooth surfaces. These surfaces are: Mesial Distal Buccal Lingual At the control appointment a suitable disclosing solution such as Bismarck Brown, Diaplac or similar is painted on all exposed tooth surfaces. After the patient has rinsed, the operator (using an explorer or a tip of a probe) examines each stained surface for soft accumulations at the dentogingival junction. When found, they are recorded by making a dash/red colour in the appropriate spaces on the record form. Those surfaces, which do not have soft accumulations at the dentogingival junction, are not recorded. After all teeth are examined and scored, the index is calculated by dividing the number of plaque containing surfaces by the total number of available surfaces. CALCULATION EXAMPLE: Assume a patient with the following plaque accumulation: Upper jaw: 34 plaque containing surfaces Lower jaw: 36 plaque containing surfaces The total available surfaces for upper and lower jaw were, in this example, 52 and 48 receptively. Plaque Index

=

(The number of plaque containing surfaces) / (The total number of available surfaces) (34 + 36) / (52 + 48) = 70 / 100 = 0.70

Meaning the plaque index for this patient is 70% at initial control appointment. Plaque Control Record

63

Another example: To determine an individual's score, the clinician multiplies the number of surfaces with plaque by 100, and divides that by the number of tooth surfaces examined. In the picture above, if an individual has 25 teeth, that equals to 100 surfaces. If 70 surfaces are found to have plaque, then 7000 is divided by 100, leaving a plaque control index of 70%. A score under 10% is considered good.

Plaque scores The patient's oral hygiene can be assessed using a plaque scoring system. A disclosing solution is applied to all of the teeth so that the plaque can be easily visualised. Always remember to apply vaseline to the lips prior to applying the disclosing solution to prevent the solution staining the lips. Then, by using a probe, the presence or absence of plaque is noted on four surfaces of each standing tooth (mesial, buccal, distal and lingual/palatal surfaces). The results are plotted on a plaque scoring chart and a percentage plaque score is calculated. This is the percentage of sites with plaque present out of the total number of sites tested. A baseline plaque score percentage should be recorded at the beginning of treatment so that any changes can be monitored after oral hygiene instruction has been given. An improvement in oral hygiene may be an important factor in deciding whether to embark on complex restorative treatment.

64

WHO periodontal probes Two variants of WHO probes are available:

CPITN-E probe Epidemiological probe - markings at 3.5 and 5.5mm.

Periodontal probe CPITN-E CPITN-C Probe Clinical Probe - markings at 3.5, 5.5, 8.5 and 11.5mm. These additional lines may be of use when performing a detailed assessment and recording of deep pockets for the purpose of preparing treatment plan for complex periodontal therapy.

Periodontal probe CPITN-C

Basic periodontal examination (BPE)

Careful assessment of the periodontal tissues is an essential component of patient management. The BPE is a simple and rapid screening tool that is used to indicate the level of examination needed and to provide basic guidance on treatment need. Please note; the BPE does not provide a diagnosis. How to record the BPE

The dentition is divided into 6 sextants: upper right (17 to 14), upper anterior (13 to 23), upper left (24 to 27) lower right (47 to 44), lower anterior (43 to 33), lower left (33 to 37) 65

All teeth in each sextant are examined (with the exception of 3rd molars). For a sextant to qualify for recording, it must contain at least 2 teeth. (If only 1 tooth is present in a sextant, the score for that tooth is included in the recording for the adjoining sextant). A WHO BPE probe is used (World Health Organisation probe). This has a “ball end” 0.5 mm  in diameter, and a black band from 3.5 to 5.5 mm. Light probing force should be used (20-25 grams). The probe should be “walked around” the sulcus/pockets in each sextant, and the highest  score recorded. As soon as a code 4 is identified in a sextant, the clinician may then move directly on to the next sextant, though it is better to continue to examine all sites in the sextant. This will help to gain a fuller understanding of the periodontal condition, and will make sure that furcation involvements are not missed. If a code 4 is not detected, then all sites should be examined to ensure that the highest score in the sextant is recorded before moving on to the next sextant. Scoring codes

0 No pockets >3.5 mm, no calculus/overhangs, no bleeding after probing (black band completely visible) 1 No pockets >3.5 mm, no calculus/overhangs, but bleeding after probing (black band completely visible) 2 No pockets >3.5 mm, but supra- or subgingival calculus/overhangs (black band completely visible) 3 Probing depth 3.5-5.5 mm (black band partially visible, indicating pocket of 4-5 mm) 4 Probing depth >5.5 mm (black band entirely within the pocket, indicating pocket of 6 mm or more) * Furcation involvement

Both the number and the * should be recorded if a furcation is detected - e.g. the score for a sextant could be 3* (e.g. indicating probing depth 3.5-5.5 mm PLUS furcation involvement in the sextant). An example BPE score grid might look like: 4 3 3* - 2 4* When to record the BPE

All new patients should have the BPE recorded For patients with codes 0, 1 or 2, the BPE should be recorded at least annually For patients with BPE codes of 3 or 4, more detailed periodontal charting is required: Code 3: record full probing depths (6 sites per tooth) in the sextant(s) where the code 3 was recorded, in addition to recording the BPE in those sextants with scores 0, 1 or 2 Code 4: record full probing depths (6 sites per tooth) throughout the entire dentition BPE cannot be used to assess the response to periodontal therapy because it does not provide information about how sites within a sextant change after treatment. To assess the 66

response to treatment, probing depths should be recorded at 6 sites per tooth pre- and posttreatment. For patients who have undergone initial therapy for periodontitis (i.e. who had pre-treatment BPE scores of 3 or 4), and who are now in the maintenance phase of care, then full probing depths throughout the entire dentition should be recorded annually Guidance on interpretation of BPE scores Interpreting the BPE score depends on many factors that are unique to each patient. The clinician should use their skill, knowledge and judgement when interpreting BPE scores. General guidance is indicated below. The BPE scores should be considered together with other factors when making decisions about whether to refer (as outlined in the companion BSP document “Referral Policy and Parameters of Care”). 0 1 2 3 4 *

No need for periodontal treatment Oral hygiene instruction (OHI) OHI, removal of plaque retentive factors/overhangs, prophylaxis, root surface debridement (RSD) OHI, RSD OHI, RSD. Assess the need for more complex treatment; referral to a specialist may be indicated. OHI, RSD. Assess the need for more complex treatment; referral to a specialist may be indicated.

Referral policy and parameters of care

Referral of patients with periodontal problems to either specialist practitioners or hospital consultants depends on several factors including: The severity of disease and complexity of treatment required. The patient's desire to see a specialist or undergo specialist treatment. The GDP's knowledge, experience and training to treat patients with a range of periodontal problems. The presence of other complicating factors such as a patient’s medical history or other  comorbidity. The referral policy here is based on a simple assessment of case complexity using the Basic Periodontal Examination (BPE) and is intended as a guideline for clinical practice. Further background information is available, including detail of BPE-based periodontal screening and related periodontal assessments, in the associated BSP policy document – “The Basic Periodontal Examination.” Parameters of Care

It is the responsibility of the dentist to monitor/screen patients regularly for the presence of periodontal diseases, including the use of relevant radiographs, to make a diagnosis and institute a treatment plan with defined therapeutic goals. On occasions a GDP may wish to refer a patient for a specialist opinion at an early stage to assist with diagnosis and treatment planning. All periodontal assessments should be recorded in patients’ clinical records. Where  treatment has been previously provided, outcome assessments should similarly be recorded. 67

The findings of every periodontal examination must be presented to the patient. Treatment options and consequences of no treatment should be explained. Even where referral for further treatment is considered, initial therapy including oral hygiene instruction and supra and subgingival scaling should normally be carried out in the primary care setting by the GDP or dental hygienist. Control of other modifiable risk factors where indicated, particularly smoking, should also be instigated by the GDP, if necessary by referral to smoking cessation services. In certain cases, for example because of the health of the patient, non-compliance, or the severity of the disease, treatment to simply prevent the progression of disease may be appropriate. In these cases initial therapy may become the end point. The GDP should organise suitable maintenance care at appropriate time intervals for treated patients, if possible with dental hygienist/dental therapist input. This is the case irrespective of whether the active treatment was provided in a primary or secondary care setting. As a guideline for referral policy using Periodontal Treatment Assessment criteria, Complexity 1 cases should generally be treated in general dental practice, Complexity 2 cases may either be treated by the GDP or referred; Complexity 3 cases should mostly be referred. It is worthy of note that sometimes apparently simple periodontal treatment may have to be delivered by Specialists as part of a more complex integrated treatment strategy. Equally, patients falling into the Complexity 3 category may not necessarily require care from a specialist. Initial nonsurgical treatment of cases should generally be carried out in general practice. Periodontal treatment assessment

Based upon the Basic Periodontal Examination (BPE) Criteria: Complexity 1 BPE Score 1 - 3 in any sextant Complexity 2 BPE Score of 4 in any sextant Surgery involving the periodontal tissues Complexity 3 Patients with BPE scores of 4 in at least one sextant, and one or more of the following factors: Concurrent medical factor directly affecting the periodontal tissues, (eg diabetes, medication, etc); Complicating root morphologies / anatomical factors; Non-response to previous optimally carried out treatment Diagnosis of aggressive periodontitis as assessed either by severity of disease for age or based on rapid rate of periodontal breakdown; Patients requiring surgical procedures involving tissue augmentation or regeneration, including surgical management of mucogingival problems; Patients requiring surgery involving bone removal (eg crown lengthening); Patients requiring surgery associated with osseointegrated implants. The presence of a relevant modifying factor increases the complexity by 1 increment, and is not cumulative. 68

Modifying factors that are relevant to periodontal treatment:

Co-ordinated medical or dental multi-disciplinary care Medical history that significantly affects clinical management (see below) Regular tobacco smoking Special needs for the acceptance or provision of dental treatment. Concurrent mucogingival disease (e.g. erosive lichen planus) Medical history that significantly affects clinical management:

Patients with a history of head / neck radiotherapy or intravenous bisphosphonate therapy. Patients who are significantly immunocompromised or immunosuppressed. Patients with a significant bleeding dyscrasia / disorder. Patients with a potential drug interaction. The index of treatment needs for periodontal treatment assessment is based on the most widely used practitioner oriented Basic Periodontal Examination (BPE) as devised by the British Society of Periodontology. It sets complexity codes in a simplistic manner with the addition of a list of modifying factors that are relevant to periodontal treatment and an outline of medical histories that significantly affect clinical management. It is strictly a complexity assessment and does not address either the motivational aspects of treatment or the prioritisation of treatment. Nevertheless it is a very useful tool not only for providing guidelines of complexity but also for indicating according to complexity where treatment may be carried out. As with all treatment involving treatment teams, the long term success of these care pathways depend on good communication between the clinicians involved to ensure consistency of treatment objectives and appropriate long term follow up.

69

Bleeding scores Bleeding on probing the gingival sulcus is associated with gingival inflammation and is therefore an important indicator of disease. Gingival bleeding is recorded at four sites on each standing tooth mesial, buccal, distal and lingual/palatal. A WHO periodontal probe is inserted to the base of the gingival sulcus at each site and moved gently along the tooth or root surface. After five seconds, the presence or absence of bleeding is noted and recorded on a chart. The bleeding score is the percentage of bleeding sites out of the total number of sites examined. A baseline measurement of the bleeding score will give an indication of the level of gingival inflammation and, when taken with subsequent bleeding scores recorded during treatment, will show any improvements in the condition of the gingival tissues following periodontal therapy.

Periodontal pocket charting A periodontal pocket is the distance in millimetres between the gingival margin and the base of the gingival sulcus and is measured with a graduated periodontal probe such a, William's probe in a healthy periodontium, the gingival sulcus is normally 1 to 2 mm in depth. Periodontal pocketing of greater than 3.5 mm is usually associated with alveolar bone loss but there can also be what is termed false pocketing, where gingival swelling and inflammation cause the gingival sulcus depth to be greater without any alveolar bone loss. The probing pocket depths are recorded at six sites on each standing tooth mesio-buccal, buccal, disto-buccal, disto-lingual or disto-palatal, lingual or palatal, mesio-lingual or mesio-palatal. The values are recorded on a chart (fig. 2.9) with the date of the examination clearly recorded. Research has shown that probing pocket depths can vary according to the thickness of the probe used, the force and direction that the probe is inserted into the sulcus and also the level of inflammation present in the gingival tissues. The recommended probing force is 25 grams but this is difficult to apply consistently unless one is using a pressure-sensitive probe with a predetermined force limit. One must therefore be aware of the limitations and accuracy of probing pocket depth measurements. Probing pocket depths should be recorded prior to any periodontal therapy such as supra- and subgingival scaling or root planing so that any subsequent measurements can hopefully demonstrate an improvement.

70

Measurement of furcation involvement If periodontal bone loss and gingival recession occurs in multi-rooted teeth, eventually the bifurcation or trifurcation area of the root will be exposed. This is significant because the furcation areas are usually inaccessible to normal plaque control methods and therefore disease in these areas is difficult to arrest. Therefore, the presence and level of furcation involvement must be recorded. Furcation sites are classified as follows: Level 1 - Horizontal loss of supporting tissues not exceeding 1/3 of the width of the tooth. Level 2 - Horizontal loss of supporting tissues exceeding 1/3 of the width of the tooth, but not encompassing the total width of the furcation area. Level 3 - Horizontal 'through and through' destruction of the supporting tissues in the furcation. Mobility assessment Progressive loss of alveolar bone will eventually lead to tooth mobility. Excessive tooth mobility can lead to pain and patient discomfort on eating. Tooth mobility can be classified as follows: Level 1 - Movement of the tooth of 0.2-1 mm in a horizontal direction. Level 2 - Movement of the tooth of greater than l mm in a horizontal direction. Level 3 - Movement of the tooth in a vertical as well as a horizontal direction.

71

It must be remembered that abnormal tooth mobility can be caused by other factors such as abnormal occlusal load or trauma. These possible factors must be taken into account in your treatment planning. Loss of attachment If periodontal bone loss is acco1-npanied by gingival recession, then there can be considerable loss of supporting tissues without there necessarily being any periodontal pocketing. Therefore some clinicians prefer to measure loss of attachment which is the distance in millimetres from the cementenamel junction to the base of the gingival sulcus.

72

RPD design Classification Classify partially edentulous ridges. (Say It’s the cast of a Kennedy’s Class 1 modification 2 arch) Unilateral free-end saddle – Class I Bilateral free-end saddle – Class II Bounded saddle – Class III Anterior bounded saddle – Class IV Designing RPD 1. If examiner is present tell him you’ll first survey the cast to identify the path of insertion. 2. Pick up a paper and colour pencils or a pen, draw the arch on paper and start with the design. 3. Outline saddle, support (rests), clasps, reciprocal arm, indirect retainer and connector 4. Colour coding: Yellow = Saddles (mesh) Red = Support (rests) Green = Retention (clasps) Blue = Bracing/Reciprocation Black = Connection 5. Mark saddles 6. Mark X for 3rd molars if absent 7. Mark support by drawing rests 8. Rests Mesial for class I and II Adjacent to saddle for class III and IV 9. Write name of the clasp and RPI system if possible. 10. Draw clasps on teeth adjacent to edentulous spaces 11. Draw bracing and reciprocation for the clasps and saddles. 12. Indirect retention – Draw imaginary clasp axis through terminal clasps, and make a perpendicular. 13. Draw a major connector joining all the units of the RPD and reason the choice of connector. 14. Write name of the major connector in addition to the drawing. 15. Review your design Types of clasps Gingivally approaching – iBar, tBar and yBar Occlusally approaching – Aker’s clasp and circunferencial or ring clasp

73

Layout for partial denture design

74

75

Surveyor Retention – resistance to displacement away of bearing tissues Direct retention – clasps and guide planes Indirect retainers – rests seats The aims for optimum retention should be to provide: Resistance along the path of displacement - achieved by the use of guide surfaces or clasps Resistance along the path of withdrawal - achieved by the use of clasps alone Support – resistance to displacement in the direction of bearing tissues Tooth support – rest seats Mucosa support – major connector and saddle Tooth and mucosa support – RPI + free-end saddle Stability (bracing) – resistance to displacement to horizontal forces (lateral movements). Avoid dentures to rock. Major connector, flanges, guide planes, reciprocal arm Stages in designing a Cobalt-Chromium framework 1. Impression 2. Study cast (avoid plaster of Paris as it is too soft. Use dental stone) 3. Surveyor 4. Design What is a dental cast? A dental cast is a positive copy of the tissues of the jaws, made in an impression, and over which denture bases or other restorations may be fabricated. What is the purpose of the primary cast? Survey to find undercuts (eyeball the cast) Denture design What is a dental surveyor? It is a paralleling instrument used in the construction of a prosthesis to locate and delineate the contour and relative positions of abutment teeth and associated structure. Parts of the dental surveyor

76

Surveyor tools Analysing rod This metal rod is placed against the teeth and ridges during the initial analysis of the cast to identify undercut areas and to determine the parallelism of surfaces without marking the cast. Graphite marker The graphite marker is moved around the tooth and along the alveolar ridge to identify and mark the position of maximum convexity (survey line) separating non-undercut from undercut areas. When surveying a tooth, the tip of the marker should be level with the gingival margin allowing the side of the marker to produce the survey line as shown in the illustration. Undercut gauges Gauges are provided to measure the extent of horizontal undercut and are available in the following sizes: 0.25 mm, 0.50 mm and 0.75 mm. 0.25mm – Co-Cr 0.50mm – Stainless steel wire 0.75mm – wrought gold By adjusting the vertical position of the gauge until the shank and head contact the cast simultaneously, the point at which a specific extent of horizontal undercut occurs can be identified and marked. This procedure allows correct positioning of retentive clasp arms on the tooth surface Wax knife (trimmer) This instrument is used to eliminate unwanted undercuts on the master cast. Wax is added to these unwanted undercut areas and then the excess is removed with the trimmer so that the modified surfaces are parallel to the chosen path of insertion. The trimming knife can also be used to prepare guide surfaces on wax patterns of crowns for abutment teeth. Path of insertion – is the path followed by the denture from its first contact with the teeth until it is fully seated. This path coincides with the path of withdrawal and may or may not coincide with the path of displacement. There may be a single path or multiple paths of insertion. Path of displacement – This is the direction in which the denture tends to be displaced in function. The path is variable but is assumed for the purpose of design to be at right angles to the occlusal plane. Purposes of the dental surveyor

77

Verify the parallelism of tooth preparations Survey the diagnostic cast Contour wax pattern for crowns Contour a crown for a milled margin Block out the master cast Surveying ceramic veneer crown Placement of internal attachment Placement of internal rest seat Machining cast restoration Surveying the master cast Facilitate RPD framework design The objectives of surveying the diagnostic cast are to Determine the most desirable path of placement that will eliminate or minimized interference Identify proximal tooth surfaces that are or need to be made parallel so that they act as guiding planes during placement and removal Locate and measure areas of the teeth that may be used for retention Determine whether tooth and bony areas of interference will need to be eliminated surgically, modification, block out or by selecting a different path of placement Determine the most suitable path of placement that will permit locating retainers and artificial teeth to the best aesthetic advantage Permit an accurate charting of the mouth preparations to be made Delineate the height of contour on abutment teeth and to locate areas of undesirable tooth undercut that are to be avoided, eliminated, or blocked out Record the cast position in relation to the selected path of insertion for future reference (Tripoding) The objectives of surveying the master cast are to Select the most suitable path of insertion by following mouth preparations that satisfy the requirements of guiding planes, retention, non-interference, and aesthetics Permit measurement of retentive areas and to identify the location of clasp terminals guided by The flexibility of the clasp arm The magnitude of the tooth undercuts The depth of the clasp terminal placed into this undercut Locate undesirable undercut areas that will be crossed by rigid parts of the restoration during placement and removal; this must be eliminated by block-out Trim block-out materials parallel to the path of insertion before duplication The surveying procedure Because surveying the master cast follows mouth preparations, the path of insertion, the location of retentive areas, and the location of remaining interference must be known before processing of the final design of the denture framework. Preliminary visual assessment of the study cast (eyeballing) - The cast is held in the hand and inspected from above to identify: The general form and arrangement of the teeth and ridge Any obvious problems noted Have an idea to whether or not a tilted survey should be employed Initial survey The cast is positioned with the occlusal plane horizontal Use the analysing rod to identify undercut areas that might be utilized to provide retention in relation to the most likely path of displacement The amount of undercut can be judged approximately from the size of the ‘triangle of light’ between  the rod and the cervical part of the tooth

78

The teeth and ridges are then surveyed with the graphite marker to register the maximum convexity (bulbosity) of the teeth An RPD can be designed on a cast which has been surveyed with the occlusal plane horizontal so that the path of insertion equals the path of displacement However, there are occasions when tilting of the cast is indicated so that the paths of insertion and displacement differ. This can be done to improve appearance, reduce interferences, improve retention, and adapt to existing guide planes If it is decided that the cast should be tilted, the analysing rod is exchanged for a marker different in colour from that used in the first survey, and the final survey is carried out. It will then usually be found that the teeth to be clasped have two separate survey lines which cross each other. In order to obtain optimum retention it is necessary to understand how to position the clasps correctly in relation to the two survey lines. The position of the survey lines and the variations in the horizontal extent of undercut associated with them should be noted The amount of undercut can be judged approximately from the size of the ‘triangle of light’ between  the marker and the cervical part of the tooth, or measured more precisely by using an undercut gauge An assessment can then be made as to whether the horizontal extent of undercut is sufficient for retention purposes After the final survey, the position of the cast in relation to the surveyor should be recorded for future repositioning if needed. The methods of recording the degree of tilt are: The tripod method The analysing rod method Surveying is undertaken to obtain information that will allow decisions to be made concerning the optimum path of insertion of the denture and also the design, material and position of clasps. The choice of a path of insertion will be influenced by The need to use guiding surfaces to achieve a pleasing appearance The need to avoid interference by the teeth or ridges with correct positioning of denture components The need to use guide surfaces for retention The decisions on these aspects of clasps can be arrived at from Measurement of the horizontal extent of undercut on abutment teeth The identification of sites on the teeth to provide reciprocation and stabilization, either from guiding surfaces or from cross-arch stabilization What can be done to change the survey line? Add composite Enameloplasty

79

80

Written OSCEs

81

Prescription writing

82

Use form FP10D, only drugs in the Dental Practitioner’s Formulary. Write legibly with ink (no cutting/over-writing) Date, full name and address of the patient, signed in ink by prescriber, age and date of birth of patient (age mandatory if under 12 yrs) Drug name in full with dose in g (for gram) for drugs over 1g, mg (for milligrams), ml (for millilitres), micrograms or nanograms (both in full) Dose frequency in full (e.g. three times daily), drugs to be taken as required (don’t write SOS),  minimum dose interval should be specified Indicate number of days of treatment in the box below D.O.B. Write number of tablets in total (quantity of drug). Write any precautions to be taken with the drug e.g. avoid alcohol for 1 week with metronidazole. Write - [No more items in this prescription] - at end instead of cutting the rest of the area. With Ibuprofen, write- Take with food Put stamp in the address box Metronidazole tablets 200 mg One tablet three times daily for 5 days Supply 15 tablets Avoid alcohol [No more items on this prescription] Amoxicillin capsules 250 mg One capsule three times daily for 5 days Supply 15 capsules [No more items on this prescription] Azithromycin tablets 500mg One tablet once daily for three days Supply 3 tablets [No more items on this prescription] Erythromycin capsules 250mg One capsule four times daily for 5 days Supply 20 capsules [No more items on this prescription] Paracetamol tablets 500mg Two tablets four times daily for 5 days, or until pain stops. Supply 40 tablets [No more items on this prescription] (paediatric 6 months to 2 years) Paracetamol oral suspension 120mg/5ml sugar-free 120mg four times daily when required Supply 100ml [No more items on this prescription] Ibuprofen tablets 400 mg One tablet three times daily for 5 days Supply 15 tablets Take with food. [No more items on this prescription] Aciclovir cream 5% Apply five times daily on the affected area for 5 days Supply 2g tube [No more items on this prescription]

83

Nystatin Oral Suspension 100,000 units/ml 1 ml after food four times daily for 7 days Supply 30 ml [No more items on this prescription] Advice: Hold in mouth near lesion for 5 minutes before swallowing. Continue use for 48 hours after lesions have healed. (Denture stomatitis) Miconazole Oromucosal gel 24mg/ml sugar free Apply a pea-sized amount to fitting surface of upper denture after food four times daily Supply 80 g tube [No more items on this prescription] (Trigeminal neuralgia) Carbamazepine 100mg tablet One tablet two times daily Supply 28 tablets [No more items on this prescription]

Fluoride prescription: Sodium fluoride 2,800 ppm toothpaste Indications: high caries risk patients aged 10 years and over. Sodium fluoride toothpaste 0.619% Apply 1cm twice daily after meals using a toothbrush Supply 75ml Sodium fluoride 5,000 ppm toothpaste Indications: patients aged 16 years and over with high caries risk, present or potential for root caries, dry mouth, orthodontic appliances, overdentures, those with highly cariogenic diet or medication. Sodium fluoride toothpaste 1.1% Two times daily Apply 2cm tree times daily after meals using a toothbrush Supply 51g

84

85

86

Inhalation Sedation Machine What checks will you perform before giving inhalation sedation to a small child? Identify the parts of the machine and list eight checks you would do before using the machine Parts of a relative analgesia machine

Machine head

87

Gas cylinders Oxygen and nitrous oxide cylinders Usually E cylinders (E size) (O2 cylinder-2200 psi 680L, N2O cylinder 720 psi) attached to the machine. Pressure gauges are attached to cylinders Valves Pressure releasing valves (regulator) Check valves Exhaust valve (pressure release valve) Oxygen flush valve Negative pressure relief valve Machine head Measure O2 or N2O in L/min Allows the anaesthetist to set the O2 or nitrous oxide flow rates Rebreathing bag (reservoir bag) Manometer (pressure measuring dial) Scavenger system Suction Nose piece/nasal hood Flow regulator/mixing valve Gas percentage regulator Emergency oxygen over-ride (oxygen flush) Checks for Sedation machine Verify backup ventilation equipment is available and functioning Check oxygen cylinder supply- One cylinder must be at least half full Check central pipeline supplies - Check for proper connection at wall. Check the pipeline pressure gauge Test flow meters Check initial status of breathing system Test ventilation systems and unidirectional valves Perform leak check of breathing system Adjust and check scavenging system Check, calibrate, set alarm limits of all monitors. Select appropriate size of nasal mask and clean with alcohol. Check automatic cut-out Check reservoir bag Check bleed system Check monitoring system, oxygen and emergency suction are available in the recovery area Check pulse oximeter is available and functioning Patient checks Check consent has been obtained Check patient understands the procedure Check pre-sedation assessment has been performed – Airway, pulse, B.P. etc. Check patients medical history and any prescribed medication and its implication Check responsible adult escort is available Check escort understands the procedure and is willing to take responsibility of the patient. Check size of nose-piece Check compliance with pre-treatment instructions - e. g. Check patient has fasted 2-4 hours prior to start Check arrangements to travel by private car or taxi and not public transport after the procedure

88

Parts of the machine Nitrous oxide flow meter Oxygen flow meter Gas mixture control Gas flow rate control

Oxygen flush button Air entrainment valve Common gas outlet

RA machine checklist (aims) To check contents of all gas cylinders To check correct functioning of controls and flowmeters To check the automatic cut-out of nitrous oxide flow To check reservoir bag for leaks Sequence of using the RA machine Start with all cylinders off Gas Check Open "FULL" oxygen cylinder Check pressure gauge rises Open "FULL" nitrous oxide cylinder Check pressure gauge rises Bleed System Open flow control with mixer control set 30% oxygen Switch off both cylinders (nitrous oxide first) Check that both flowmeters fall to zero Gas Check Open both "in use" cylinders Check that both pressure gauges rise Flowmeter and Controls Check Set mixer control 100% oxygen Open flow control. Set flowmeter at 6 litres/minute oxygen Set mixer control at 50% Check that both flowmeters indicate 3 litres/min Automatic cut-off check

Turn off the oxygen cylinder - wait Check that both flowmeters fall to zero Reservoir Bag Check Turn the oxygen cylinder back on Turn off flow control. Set mixer dial at 100% oxygen Occlude common gas outlet Press oxygen flush to inflate reservoir bag and check for leaks

Machine ready for use Emergency and monitoring equipment Site of emergency equipment known Site of oxygen known Positive pressure ventilating bag Suction - mobile back-up

Dental unit Pulse oximeter Blood pressure monitor Emergency drugs - available and in date

Patient check Patient/parent understands what is planned Written consent obtained Medical history checked Normal medication checked

Last meal/drink Escort and transport Weight recorded BP recorded

89

Standards in Conscious Sedation for Dentistry http://www.dstg.co.uk/teaching/standards-oct-02 Introduction Pain and anxiety management of patients is paramount in dentistry. Conscious Sedation is a fundamental part of the pain and anxiety management of patients in operative dentistry. All patients need and deserve to expect appropriate and individually considered pain and anxiety control for any dental procedure. Properly provided Conscious Sedation is safe, valuable and effective for dental patients. It is of key importance to maintain a wide margin of safety between Conscious Sedation and the unconscious state of general anaesthesia where verbal communication with the patient or protective reflexes are lost. It is inappropriate to restrict clinical techniques as long as the the patient remains in verbal contact with the sedationist. It is misguided to believe that single drug or multiple drug techniques are inherently safer or appropriate for all cases. Training and experience are essential. Every practitioner providing Conscious Sedation should gain the theoretical and practical hands on, supervised, clinical training necessary to practise the individual techniques for the safe management of the dental patient. The following guidelines are intended to assist in the practice of Conscious Sedation. These recommendations are directed to all practitioners providing Conscious Sedation for dentistry wherever the setting. Their purpose is to ensure that Conscious Sedation for dentistry continues to be as effective and safe as possible. The techniques described in this document are appropriate for use bye an operator sedationist whereby the person providing the dentistry also delivers the Conscious Sedation. Background During the last decade there has been a number of reports relating to the provision of general anaesthesia and Conscious Sedation for dentistry from advisory bodies, the Departments of Health and specialist societies. In 1990 the report ("General Anaesthesia, Sedation and Resuscitation in Dentistry" prepared by the Standing Dental Advisory Committee") heralded considerable change in the provision of these services for dentistry. (appx.1) A report by the Clinical Standards Advisory Group entitled "Dental Genreal Anaesthesia" in 1995 recommended standards for patient care, safety facilities and training for dentists and anaesthetists and supporting staff. (appx.2) During this time the Society for the Advancement of Anaesthesia in Dentistry and the Association of Dental Anaesthetists issued specific guidelines in relation to contemporary clincical practice both in relation to General Anaesthesia and Conscious Sedation. (appx.3&4) There were two reports from the Royal College of Surgeons of England in 1993 and 1996. (appx.5&6) More recently The Dental Sedation Teachers Group have published curriculum guidance for undergraduates and the competent graduate. (appx.7&8) In 1998 the General Dental Council recognised the views of the specialist societies and the Royal Colleges. It endorsed the need for Conscious Sedation provision rather than the continuing provision of General Anaesthesia as a demand led service. This has reduced the use of General Anaesthesia in primary dental care. (appx.9) In July 2000 the publication of "A Conscious Decision", a report by a group chaired by the Chief Medical Officer and Chief Dental Officer of England, finally heralded the removal of General Anaesthesia associated with dentistry from non hospital settings. (appx.10) This report made a number of recommendations concerning Conscious Sedation. The government has accepted the report. A watershed has been reached where the concept of conscious Sedation for dentistry is no longer considered alongside that of General Anaesthesia.

90

In July 2000, an expert group representative of all branches of dentistry was convened by the Society for the Advancement of Anaesthesia in Dentistry charged to consider standards for Conscious Sedation in dentistry. This document aims to identify good clinical practice, which is appropriate to both NHS and private dental care both within and outside hospitals. The following guidance will help attain and maintain high clinical standards, standards which all patients deserve. Definition of Conscious Sedation Our definition of Conscious Sedation is: a technique in which the use of a drug or drugs produces a state of depression of the central nervous system enabling treatment to be carried out, but during which verbal contact with the patient is maintained throughout the period of sedation. The drugs and techniques used to provide conscious sedation for dental treatment should carry a margin of safety wide enough to render loss of consciousness unlikely. The level of sedation must be such that the patient remains conscious, retains protective reflexes, and is able to understand and respond to verbal commands. This definition was originally proposed in the Wylie Report (1978),(appx.11) and has been adopted by the General Dental Council, The Department of Health, The Society for the Advancement of Anaesthesia in Dentistry, The Dental Sedation Teachers Group, The Scottish Office National Dental Advisory Committee and the British Society of Gastroenterology. The above definition describes the state of Conscious Sedation, and does not attempt to prescribe how this is achieved. Specifically, it is acknowledged that many different techniques, involving the use of one or more drugs and varying routes of administration, will fulfil this definition provided that there is an adequate margin if safety. Any technique resulting in the loss of consciousness or abolition of protectice reflexes is defined as General Anaesthesia. There is no place for the practise of General Anaesthesia called Conscious Sedation. Educational & Training Standards Education and training in Conscious Sedation needs to ensure that ALL members of the dental team providing treatment under Conscious Sedation have received theoretical, practical and clinical training before undertaking independent practice. Theory All the topics referred to in this document must be included. Practical Skills Drugs and Equipment: Practical Training in the use of drugs and equipment used to provide Conscious Sedation and monitoring during Conscious Sedation is essential for sedationists and sedation assistants before they progress to clinical training. Complications: Training in the management of Conscious Sedation-related complications, in addition to the standard requirement for proficiency in Basic Life Support, is essential for all clinical staff (sedationist/assistant/operating dentist) and is desirable for non-clinical support staff. Basic Life Support must conform to contemporary guidelines issued by the Resuscitation council (UK) (appx.13) and General Dental Council Guidance to Dentists on Professional and Personal Conduct (Maintaining Standards). Clinical Skills Supervised hands-on experience must be acquired by both sedationists and their assistants for EACH Conscious Sedation technique used. This may be provided in a variety of settings.

91

The method and timespan allowed for acquisition of this supervised practice may vary depending upon local circumstances. The minimum number of documented supervised ases completed should be no less than those specified by appropriate authorities and updated in line with changing recommendations. For example 'The Competent Graduate' [Dental Sedation Teachers Group] gives guidance for undergraduate dental students. (appx.7) Sedation assistants/nurses may follow the requirements of the 'Log of Practical Experience' for entry to the National Examination Board for Dental Nurses examination for the Certificate in Dental Sedation Nursing. Provision of Education and Training This can be provided in-house in clinical areas where Conscious Sedation is practised and/or in more formal courses. Those arranging such training fro their staff have a duty to ensure that the quality of training and trainers is appropriate and that all theoretical and practical training is documented. Retention and improvement of knowledge and skill relies upon regular updating by means of refresher courses and/or a programme of continuing assessment or review as a routine practice activity. The interval at which such updating is required will depend upon local circumstances but must be carried out at least once a year. Updating may also be undertaken before recommencing practice after a break of 12 months or more. All Education and Training programmes must be updated regularly and based on currently accepted standards as promulgated by appropriate authorities. Examples of bodies who publish these are listed as Appendix 1. Environment for Sedation The Dental surgery should be large enough to allow adequate access for the dental team all around the patient. As with all dental techniques the dental chair must be capable of being placed in the horizontal position. Equipment for Inhalation Sedation Dedicated purpose-designed Relative Analgesia machines for dentistry should be used. Such machines should conform to British Standards (appx.14) and be maintained according to manufacturers' guidance with regular, documented servicing. Gas supply lines for Relative Analgesia machines must be connected by non interchangeable colour coded pipelines. On installed pipelines there must be a low pressure warning device and an audible alarm. Nitrous oxide and oxygen cylinders must be stored safely with regard to current regulations. Cylinders must be secured safely to prevent injury. There should be adequate scavenging of waste gases where inhalation sedation is used since inadequate scavenging may result in unacceptable risks to health of the dental team. Adequate scavenging of gases should not rely on window opening or air conditioning alone and it should conform to current COSHH standards. (appx.15&16) Breathing systems should have a separate inspiratory and expiratory limb to allow proper scavenging. Nasal masks should be close fitting providing a good seal without air entrainment valves. Equipment for Intravenous Sedation All the appropriate equipment for the administration of intravenous sedation must be available in the surgery including syringes, needles, cannulae, surgical wipes/tapes/dressings, tourniquets and labels. Purpose-designed, calibrated and appropriately maintained equipment is required for all infusion techniques. It is mandatory to be able to administer supplemental oxygen or oxygen under intermittent positive pressure ventilation to the patient should the need arise. This must conform with General Dental Council guidelines.

92

Patient Assessment and Selection Careful and thorough assessment of the patient ensures that correct decisions are made regarding the planning of subsequent treatment. The full spectrum of patient management techniques should be explored. An objective assessment and discussion with the patient enable decisions to be made such that the patient receives the most appropriate type of Conscious Sedation, administered in the correct environment by the sedationist best able to provide the service. Reasons for Requiring Conscious Sedation The patient should be asked directly to state their anxieties about dental treatment and what precipitated them. A discussion of the proposed treatment choices and the patient's attitude to them should be investigated. Documented History All dental patients should have a thorough medical, dental and social history taken and recorded for each course of treatment. Clinical Assessment After a dental examination a provisional treatment plan can be formulated. Assessment of the patient's general fitness including colour, pulse and respiration is important in patient selection. Blood pressure measurement is an essential part of risk assessment for sedation. The American Society of Anesthesiologists (ASA) Physical Status classification should be determined. (appx.18) There are few absolute contraindications for Conscious Sedation. Relative contraindications are important however and can only be considered in the light of the full knowledge of the case which thorough assessment provides. Preparation of Patients for Conscious Sedation Patients who are scheduled to receive Conscious Sedation must receive careful verbal and written instructions as to the effects of the sedation and their responsibilities prior to the sedation appointment. Before a Conscious Sedation appointment patients should be advised to take only light food. Starvation for Conscious Sedation is undesirable. Specific written consent must be obtained from all patients who are to receive treatment under sedation. A responsible adult escort must accompany the patient home friom the dental surgery and assume responsibility for the patient's post-sedation care. Both patients and escort must understand and accept that this responsibility is delegated to the escort and both must agree to comply with this. It is therefore essential that both patient and escort clearly understand the effects of sedative agents and the consequences of failing to follow all post-sedation instructions. Wherever possible arrangements should be made for the patient and escort to travel home by private car or taxi rather than public transport. Where this is not possible, the escort must be made aware of the added responsibilities of caring for the patient during the journey home. If, in the opinion of the sedationist, either the patient or the escort appear to be unwilling or unable to comply with these requirements, Conscious Sedation should not be given. With the exception of adult patients receiving nitrous oxide / oxygen inhalation sedation, an escort is mandatory for conscious sedation. (appx.8) It is desirable that the escort should look after the patient for the rest of the day or has made reliable arrangements for a capable adult to undertake that care. Thus the provision of conscious sedation may be unsuitable for a patient who lives alone or who solely cares for children, elderly and/or dependent relatives.

93

The Consent Process Consent is a communication process. The General Dental Council currently requires that where sedation is provided then the patient should also provide written consent. It is important to remember that a signature on a form can be misleading and the mere presence of such a signature does not guarantee that the consent obtained is valid. "Consent is the voluntary and continuing permission of a patient to receive a particular treatment. It must be based upon adequate knowledge of the purpose, nature and likely effects and risks of that treatment, including the likelihood of its success and any alternative to it." The law provides for all persons ages 16 and over to consent to dental treatment. In order to provide valid consent a patient must be able to comprehend the information provided, retain it and assimilate the same information so as to be able to make a decision. Caution must be applied where children under 17 provide consent when they are competent to do so. Patients who are already sedated are unlikely to be competent to take decisions regarding consent for treatment. It is therefore inappropriate to try to seek consent for dental treatment from a patient who is sedated. All decisions made by patients in respect of sedation should be voluntary. Patients should not be coerced in any way to accept sedation techniques if they do not wish to do so. Sedation should be presented as an option in anxiety control with other options being pointed out to the patient. If treatment plans cannot be pre-determined this should be explained to patients with an explanation, in broad terms, of the possible treatment. Patients should be given an opportunity to ask about all aspects of their treatment with questions answered truthfully. Records and Documentation The most accurate, contemporaneous record of the consent process is the clinical record card. It is recommended that a clinical record for conscious sedation should include the reason for Conscious Sedation and evidence of the consent process. The consent process includes both sedation and the treatment to be provided including: Written medical history Previous dental history. Written instructions have been provided pre- and post-operatively. The presence of an accompanying adult. The patient has complied with pre-treatment instructions. The medical history has been checked and acted upon. Records of drugs employed, dosages and times given including site and method of administration. Previous Conscious Sedation / General Anaesthetic History. Pre-sedation assessment. Any individual specific patient requirements. Suitable supervision has been arranged. There is written documentation of consent for sedation (Consent Form). Records of monitoring techniques. Full details of dental treatment provided. Post sedation assessment. Aftercare Recovery Recovery from sedation is a steady progression from completion of treatment through to discharge into the care of an escort. The first stage of recovery is normally in the dental chair. Once the patient is recorvered sufficientyl to move to a resting area, they should be carefully guided and supported.

94

This recovery area should be serparate from the main waiting room and contain furniture and firments for the comfort and well being of the patient. A member of the dental team should supervise the patient during this period. Equipment and drugs for dealing with medical emergencies must be available. The dentist or sedationist must be available to see the patient urgently to deal with any problems that may arise. Discharge The decision to discharge a patient into the care of the escort following any type of sedation must be the responsibility of the dentist or the sedationist. The patient must have a responsible adult as an ascort. The criteria for discharge are that the patient should be able to walk unaided without stumbling or feeling unstable. Aftercare Instructions In addition to specific instructions given to the patient and escort regarding the sedation, details of the dental treatment provided should be given to patient and and escort including any aftercare arrangement relating to pain control and possible postoperative bleeding. Adequate information including how to contact the dentist if necessary must be given. Conscious Sedation Techniques Introduction The three standard sedative techniques used in dentistry (inhalation, oral and intravenous) all work for the vast majority of patients. Transumucosal and intranasal techniques require special training and experience. These forms of treatment should be restricted to the experienced sedationist in an appropriate environment. The technique that is chosen must be tailored to provide the most appropriate anxiety relief for the individual patient. There are a few absolute guidelines. As a general rule the simplest technique should be used in all cases. No one technique will be successful for all patients whether it be the use of a single drug, or the use of multiple agents. In certain situations it is possible to mix techniques; for example for a needlephobic patient it may be that inhalation sedation is used initially to allow intravenous cannulation. All drugs must be carefully labelled even when only one drug is drawn up. All syringes in use in the surgery must be labelled, whether containing dental medicaments (e.g. root canal medicaments) or drugs (e.g. intravenous sedatives). Proper checking and labelling of all drugs is particularly important especially as drugs are often used in different concentrations. There must be adequate safeguards where the same drugs of different concentrations are stored in that no confusion occurs over the choice of drug. Drugs should be given according to accepted administration/titration protocols. Monitoring: Monitoring standards must be stringent for all types of Conscious Sedation. Clinical monitoring of colour, pulse and respiration is of particular importance throughout all Conscious Sedation procedures. (appx.3) Surgery staff assisting with Conscious Sedation procedures must be capable of monitoring the clinical condition of the patient. For inhalation sedation clinical monitoring of patient without further electro-mechanical devices is adequate. Monitoring for intravenous sedation must include the proper use of pulse oximetry and blood pressure monitoring. Inhalation sedation:

95

The only currently recommended technique for inhalation sedation is the use of a titrated dose of nitrous oxide with oxygen. Intravenous sedation: The standard technique used for intravenous sedation is a titrated dose of a single benzodiazepine. Use of a continuous infusion of propofol has gained some popularity in recent years. Other drugs or combinations of drugs may be required in specially selected circumstances. These forms of treatment should be restricted to the experience sedationist in an appropriate environment. In both Inhalation Sedation and Intravenous Conscious Sedation, success is due to titrating the dose given to the patient's needs. Fixed doses or bolus techniques are unacceptable. Oral / Intranasal / Transmucosal Sedation: These are techniques where doses of a drug are administered to the patient under the direct supervision of the prescribing dentist with the aim of achieving a satisfactory level of sedation for dental treatment to be carried out. This is different from pre-medication where small doses of drug are given to aid the patient's journey to the surgery, or to assist in a restful night's sleep before the appointment. Drugs currently used to produce these types of Conscious Sedation include midazolam and temazepam. Appropriate procedure need to be followed for agents not licensed for oral, intranasal or transmucosal use. (appx.19&20) Conscious Sedation for Children Any person under the age of sixteen may be considered for clinical purposes, to be a 'child'. If a child of any age is unwilling or incapable of co-operation they are not suitable candidates for Conscious Sedation. Practitioners must be aware that there circumstances where Conscious Sedation is inappropriate. In these cases referral for general anaesthesia must be considered. Children have different requirements and responses. Conscious Sedation for children should only be undertaken by teams which have training and experience in the case selection, behavioural management, and administration of sedation for this age group, and in an appropriate environment. Provided that this requirement is fulfilled, there is no contraindication to the administration of Conscious Sedation to children in the Dental Practice setting. Inhalation Sedation Nitrous oxide / oxygen Conscious Sedation should be the first choice for paediatric dental patients who are unable to cope with local anaesthesia alone and have a sufficient level of understanding to accept the procedure. Nitrous oxide inhalation sedation should be offered to children with mild to moderate anxiety to enable them to better accept treatment and to facilitate coping across sequential visits. Nitrous oxide inhalation sedation sedation can be used to facilitate dental extraction in children and is the method of choice for anxious children who undergo elective orthodontic extractions. (appx.21&22) Intravenous Sedation Topical anaesthetic should be used prior to local anaesthesia and at the cannulation site. There is insufficient scientific evidence in the United Kingdom to support the routine use of intravenous sedation in children for dentistry. Oral / Intranasal / Transmucosal Sedation These forms of treatment should be restricted to the experienced sedationist in an appropriate environment. Complications

96

Any Conscious Sedation procedure may be associated with complications. It is vitally important that the incidence is kept to the lowest possible leve. The management of complications requires the whole dental team to be: aware of the risk of the development of complications. appropriately trained and regularly rehearsed in emergency procedures. fully equipped with appropriate means of airway protection, oxygen delivery and drugs for emergency use. It is essential that the equipment is carefully checked, that the oxygen supply is secure and adequate and that the drugs are in-date with all requisite means for their administration. It is vitally important for the whole team to be prepared and to rehearse the routine regularly. Clinical Governance and Audit It is a requirement of good practice that all professional clinicians work with colleagues to monitor and maintain awareness of the quality and care that they provide for their patients. This is a basic principle of clinical governance and risk management. Attention must be given to risk awareness, risk control, risk containment and risk transfer. Evidence of active participation in personal clinical audit in an essential feature of clinical governance.

97

IOTN Question 1 IOTN + referral to orthodontist. Props provided: IOTN ruler, dental casts, headed paper and lists of IOTN scores given. It was IOTN of 4 or 5 as there was only one premolar missing but not sure if impacted or extracted as no x-ray was given. Write a referral letter with malocclusion - it was class 2 division 2 Question 2 Cast & IOTN ruler was kept, the Dental Health Component Sheet & Aesthetic Component picture charts were provided and questions asked such as: IOTN – Index of orthodontic treatment need What is the dental Health Component Score for this patient? (5) Which point on the sheet gives this score? (a) What is the orthodontic treatment need in this patient? (Great need) What is the use of the aesthetic component score? (It is used to compare the attractiveness of dentition on a Grade of 1 to 10. Also, if Dental Health Component is 3, and Aesthetic Component lies between 6 to 10, Orthodontic treatment is needed. It is usually checked only when D.H.C. is 3) Any other indices used in Orthodontics? (Pear Assessment Rating – PAR) Method of measurement of the dental health component To ensure consistency in evaluating malocclusions, the dentition must be assessed in a systematic (hierarchical) way. Firstly, always check that all the teeth are present as teeth may be impacted (5i) or that they did not develop (5h or 4h). If there are no missing teeth the overjet should be measured. However, if the overjet is less than 6mm the deviation of the lower dentition resulting from a crossbite should be recorded. The displacement of contact points should be recorded that are greater than 2mm and overbite and open bites falling into Grades 3 and above. This sequence can be remembered by the mnemonic “MOCDO”. (MOCDO) Missing teeth (5i, 5h, 4h) Over jet (5a, 4a, 3a, 2a / 5m, 4m, 4b, 3b, 2b) Cross bite (4c, 3c, 2c) Displacement of contact point i.e. Crowding (4d, 3d, 2d) Over bite /Open bite (4f, 3f, 2f / 4e, 3e, 2e) Abbreviations i - incompetent lips c - competent lips

O.B. - overbite G + P – gingival and palatal trauma

Dev – deviation Interdig – interdigitation

Check the cast for the defects in the given order. Only the worst anomaly is recorded. Then check its extent with the IOTN ruler and compare with the IOTN Dental Health Component and give the grade. First check if all the teeth are present as teeth may be impacted (5.i) or did not develop (5.h or 4.h). If no missing teeth, measure the overjet. If it is less than 6mm, deviation of lower dentition resulting from a crossbite is recorded. The displacement of contact points greater than 2mm is checked next and overbite and open bites falling into Grades 3 and above. ONLY THE WORST OCCLUSAL ANOMALY IS RECORDED. Reverse over jet = Cross bite in anterior teeth Aesthetic Component is also called Scan

98

99

Dental Health Component of IOTN

A ruler can used to record the DHC. Disposable or autoclavable DHC rulers are available

100

Aesthetic Component of IOTN The patient can also be referred for orthodontic treatment when the dental health component is borderline grade 3 if the aesthetic component is above 5, falling in the right hand column (green). This is when the aesthetic component is employed.

101

102

Lichen planus and lichenoid reaction Lesion Reticular lichen planus or lichenoid reaction Clinical subtypes of lichen planus(6 types) Reticular - white lacy streaks (Wickham's striae) Bullous Erosive atrophic, popular plaquelike Predisposing factors Drugs, eg antimalarials, antidiabetic drugs, NSAIDs, gold salts, antihypertensives, metformin (diabetes), penicillamin (arthritis) Dental restorative materials, eg amalgam and gold Graft-versus-host disease Hepatitis C and chronic liver disease Skin, nails and genitalia may be affected Itchy, purple, raised patches are often seen on the wrists. They often occur following trauma (the Koebner phenomenon) If the skin of the scalp is affected, alopecia or hair loss may be seen Nails appear ridged when affected. Genitalia show white lesions similar to those seen in the mouth Symptoms (5 points) White lacy streaks on the inside of cheek or on the tongue on both sides. Small red areas on the gums Itchy skin rash on the inside of hands Difficulty in eating and toothbrushing Soreness or burning sensation in the mouth Red, open sores in the mouth Scaly patches

Clinical features Bilaterally symmetric white lesions on buccal mucosa or dorsum of tongue – Wikham’s striae Various clinical presentations – Reticular, Plaque, Erosive, Bullous, Striae, Papular Desquamative gingivitis Lesions on flexor surface of forehands and wrists – Purplish papules 2-3 mm with glistening surface and straie Lesions on scalp and nails Soreness or burning sensation in the mouth

Types of lichen planus Reticular, Plaque, Erosive, Bullous, Striae, Papular Differential Diagnosis (5 points) Lichenoid drug reactions Dysplastic leukoplakia

103

Discoid Lupus Erythematosus Chronic ulcerative gingivitis Erythroplakia Graft Vs Host Disease Etiology is unknown, other possible causes are: Drugs Antihypertensives – Beta-blockers, Captopril, Thiazide diuretics NSAIDs Gold salts Oral hypoglycaemic Allopurinol Antimalarials Methyldopa Penicillamine Tricyclic antidepressants Reaction to restorative materials e.g. Amalgam or gold Liver disorders esp. Hepatitis C Graft vs Host disease Treatment Biopsy and blood test to confirm diagnosis Corticosteriods Triamcinalone dental paste Beclomethasone aerosols If topical treatment fails, referral to specialist followed by systemic corticosteroids or steroid-sparing immunosuppressant

104

Denture stomatitis Picture of denture stomatitis shown and questions asked What is this condition? Denture stomatitis / Chronic atrophic candidosis Name of the causative organism Candida albicans Give other possible cause for the lesion. Mechanical and chemical irritation Bacterial infection (Staphylococcus aureus) Allergy to denture base Why does it happen? Poor denture hygiene Ill fitting dentures Immunosuppression Overnight wear of denture Dry mouth (Xerostomia) Diabetes Antibiotics therapy Corticosteroids Iron and vitamin deficiency Smoking What other lesion is commonly associated with it? Angular cheilitis What are the predisposing factors? Local factors Xerostomia from drugs or radiotherapy Antibiotic therapy, particularly broad-spectrum agents Corticosteroids Heavy Smoking Dental appliances Systemic factors Poorly controlled diabetes mellitus (neutrophil leukocyte defects) Extremes of age Nutritional deficiencies – Iron, Vitamin B12, Folic acid Immunosuppressive drugs – corticosteroids, cytotoxic chemotherapy Immunodeficiency – Hereditary / Acquired (HIV) Is it more common on maxilla or mandible and why? It is more common in maxilla. Reasons are: Broader mucosal coverage in maxilla Cleansing action of saliva in the mandible Better suction in upper denture gives closer proximity between mucosa and denture Management Advice patient regarding denture hygiene Treatment of causative factors Treatment with medication

105

Advice Leave the dentures out for at least 6 hours in a 24 hour period to promote healing of the gums. If the gums are inflamed, leave out for longer. Clean dentures by brushing, and then soak them in a disinfectant solution overnight. Allow the dentures to air-dry after disinfection — this also kills adherent candida on dentures. Brush the mucosal surface regularly with a soft brush. See a dentist to correct ill-fitting dentures Prescribe an anti-fungal drug along with denture hygiene measures Topical miconazole or nystatin is recommended as first-line of treatment Fluconazole is recommended as second-line of treatment if oral candidosis persists. Denture stomatitis can be treated effectively by local measures (see below). However, antifungal agents can be used as an adjunct to these local measures, particularly to reduce palatal inflammation before taking impressions for new dentures. Chlorhexidine mouthwash is also effective against fungal infections. Local Measures – to be used in the first instance Advise the patient to: Brush the palate daily to treat the condition; Clean their dentures thoroughly (by soaking in chlorhexidine mouthwash or sodium hypochlorite for 15 minutes twice daily (note that hypochlorite should only be used for acrylic dentures) Leave their dentures out as often as possible during the treatment period If dentures themselves are identified as contributing to the problem (ill fitting), ensure the dentures are adjusted or new dentures are made to avoid the problem recurring Drug therapy Miconazole Oromucosal Gel, 24 mg/ml (sugar-free) Send: 80 g tube Label: Apply a pea-sized amount to fitting surface of upper denture after food four times daily Advise patient to remove upper denture, apply gel sparingly to fitting surface and then reinsert Advise patient to continue use for 48 hours after lesions have healed Do not prescribe miconazole for patients taking warfarin or statins Fluconazole Capsules, 50 mg Send: 7 capsules Label: 1 capsule daily Fluconazole can be administered for a maximum of 14 days for the treatment of denture stomatitis. Do not prescribe fluconazole for patients taking warfarin or statins. If fluconazole and miconazole are contraindicated, an appropriate regimen is: Nystatin Oral Suspension, 100,000 units/ml (sugar-free) Send: 30 ml Label: 1 ml after food four times daily for 7 days Advise patient to remove dentures before using drug, rinse suspension around mouth and then retain suspension near lesion for 5 minutes before swallowing. Advise patient to continue use for 48 hours after lesions have healed.

106

Referral letter In the exam, a referral should be brief enough to be done in 5 minutes. It should include: Upper right corner of the paper - name of the referring general dental practitioner (your name), name of the practice, its address including postcode and telephone number. Below, on the left corner - Name and address of the consultant. Below - patient's details – name, date of birth, address (including postcode), telephone number Dear… Degree of urgency and reason. Whether for advice, treatment or second opinion. Nature and history of the problem including: A clear statement of the specific dental problem for which you are referring Duration of the problem Tooth/teeth in question must be clearly stated An overview of the treatment provided already to the patient Specific problems encountered when treating the patient and details of any specific factors that may influence the extent of disease present Relevant medical history and Patient's medical practitioner details Date of referral. Points which can be added, but not important Number of teeth present. Oral hygiene status basic periodontal examination (BPE) scores Radiographs where relevant Marking scheme

Notes for examiner Order is less important here - a clear, logical structure is the point we are trying to make here, as well as the formalities of writing a sound referral letter. If you cannot read the letter then the candidate will fail the station. I would suggest the pass mark should be 8, if legible.

107

--------------------------------------------------------------------------------------------------------------------------------------Sample 1 Candidate instruction You are to write a referral letter, to your local Oral and Maxillofacial Consultant, Mr Lowry, concerning one of your patients. Patient details Mrs Janice Taylor 69 Toc!dy Lane Spingfield Chehnsford CM7 99HD DOB: 13.01.1949 / TEL No: 028655 378333 She has been coming to you over the past 4 weeks with a 2cm x 1 cm ulcer on the side of the tongue. This has not responded to any topical sprays or creams, filing down of adjacent teeth or a short course of Amoxil that you prescribed for her a week ago. She is a noninsulin dependent diabetic, for which she takes Metformin. There is nothing else of note. She has undergone an appendicectomy in 1973. She smokes 25 cigarettes per day and drinks a glass of wine each night. She is widowed and lives on her own. The ulcer looks red and inflamed, with everted edges. Sample 2 Candidate instruction You are required to write a referral letter, to your local Oral and Maxillofacial Consultant, Mr SteinanFaulkner, at the Grand Hospital, E19DD, concerning one of your patients who requires extraction of a disto-angularly impacted and partially erupted wisdom tooth. Your Address Perfect Smiles Dental Clinic 21 Chichester Way, London W14QU

Patient details Mrs Janice Taylor 69 Toddy Lane London EC1A 2PP DOB: 13.01.1983 TEL No: 07785 378333

Mrs Taylor is a 27 year old patient who has come to you with pain and swelling associated with her lower left wisdom tooth. The symptoms began about 5 days ago. This is the third such presentation in 12 months. Previously, symptoms settled with 5 day courses of metronidazole. She attends regularly otherwise. On examination, she has no facial swelling and no trismus. The LL8 is partially erupted, and distally impacted and pericoronal tissues inflamed. There is no evidence of puss collection. The dentition is otherwise sound, and there is nothing else to note other than food debris around the LL8. A periapical radiograph only shows the crown of the tooth, confirming the disto-angular position. She is otherwise fit and well, but allergic to penicillin. She takes no medication and has never had a GA before. She does not smoke and drinks 6 units a week. You have advised salty mouthwashes and prescribed metronidazole 400mg three times daily for 5 days. You think she needs a surgical removal under GA and you have agreed to refer her. ---------------------------------------------------------------------------------------------------------------------------------------

108

Sample 3

Referral letter for extraction of LL8 for a patient who has had 2 episodes of pericorinitis. Refer to NICE guidelines. ---------------------------------------------------------------------------------------------------------------Dr. A. N. Other, BDS The Dental Practice, Newham ER7 5HZ Tel. 202 8456 7865 Dr. Chopper, Thames Road Dental Practice, London ET5 OS1 Re: Mr Thomas Smith, DOB 13 Aug. 1971, 54, Burntash Avenue, Thamestown, Ph. 12345678, tel. 020 3456 7892 Dear Mr. Chopper, I would be grateful if you could see and treat Mr Smith. He reported to my surgery today with pain and swelling of the pericoronal tissues in relation to his LL8 and also with difficulty opening his mouth. On examination, his temperature was raised to 38° and neck nodes were swollen. Intra oral examination and radiographs confirm pericoronitis in relation to his impacted LL8. I have prescribed a three-day course of metronidazole 200mg and Ibuprofen 400mg to clear the infection and reduce pain and swelling. Mr Smith had similar two episodes of pericoronitis with the same tooth before, which required antibiotics. Therefore, in accordance with NICE guidelines, extraction of his LL8 is recommended. Mr. Smith is (medically fit and well) or (suffers from hypertension and diabetes which are well controlled with medication). He reports to consume 4-5 units of alcohol a week and smoke 3-4 cigarettes a day for the last 10 years. Please find the relevant radiographs attached. Yours sincerely, ___________________ Dr. A. N. Other, 7TH October 2008 ----------------------------------------------------------------------------------------------------------------

109

Sample 4 Referral letter for suspected malignant ulcer on side of tongue – Urgent referral.

------------------------------------------------------------------------------------------------------------Dr. A. N. Other, BDS, The Dental Practice, Newham ER7 5HZ Tel. 202 8456 7865 Dr. Chopper, Thames Road Dental Practice, London ET5 OS1 Re: Mr. Thomas Smith, DOB 13 Aug. 1971, 54, Burntash Avenue, Thamestown, Ph. 12345678, tel. 020 3456 7892 Dear Mr. Chopper, I would be grateful if you could see and treat Mr Smith urgently. He presented to my practice two weeks ago complaining of a sore area on his tongue which had been present for four weeks. Examination revealed no palpable neck nodes. Intraorally, there was a 1cm ulcer present on the right side of his tongue adjacent to a broken down lower second permanent molar on which I placed a temporary restoration to prevent further trauma to the area. At his review appointment today the ulcer is still present and appears to have raised rolled margins. I am suspicious of squamous cell carcinoma of the right side of tongue. Mr. Smith is medically fit and healthy. He smokes 20 cigarettes per day for the last 20 years and consumes 4-5 units of alcohol per week as beer. Yours sincerely, _______________ Dr. A. N. Other, 7TH October 2008

----------------------------------------------------------------------------------------------------------------

110

Sample 5 Orthodontic referral: ---------------------------------------------------------------------------------------------------------------Dr. A. N. Other, BDS, The Dental Practice, Newham ER7 5HZ Tel. 202 8456 7865 Dr. Jones, Consultant Orthodontist, St John's Hospital, Chelmsford. Re: Miss Jane Smith, DOB: 31/01/91 10 Main Street, Danbury CM3 7YR, Tel. 020 3456 7892 Dear Dr Jones, I would be grateful if you could see Jane as she is very unhappy about the appearance of her upper anterior teeth which are crowded. Examination reveals a class 2 division 1 incisor malocclusion with a 3mm overjet and a complete but atraumatic overbite. She has an IOTN grade 4d dental health component related to her crowded upper canines. The lower arch seems to be well aligned. Jane keeps a good oral hygiene and has no caries. She suffers from asthma which is well controlled by the use of Ventolin inhaler. I would appreciate your opinion on the management of Jane’s malocclusion. Yours sincerely, ________________ Dr. A.N. Other 7TH October 2008

----------------------------------------------------------------------------------------------------------------

111

Sample 6 --------------------------------------------------------------------------------------------------------------------------------------Dr Ortho Orthodontic Centre Next to Somewhere TZ$7LZ 21 February 2010 Dr ABC - General Dental Practitioner The dental practice Somewhere TZ4 7LY Re: Ann Patient: DOB: 21.2.1998, Tel: 0209 456 3413, The Cottage, Somewhere, TZ4 7LY Dear Dr Ortho, I have seen this pleasant 12 year old girl today who was accompanied with her mother for a review appointment. Ann complained of her top teeth sticking out and this has been of concern to her and her parents for the past 1 year. On examination, Ann presents with Class II Div 1 incisor relationship on a skeletal II base and average vertical proportions. She is in permanent dentition. She has an excellent standard of oral hygiene, has not had any previous history of dental treatment though is highly motivated towards receiving orthodontic treatment. Medically, she is fit and healthy. I would be most grateful if you could please see Ann to assess her concern and possible need for orthodontic treatment. Please find enclosed copies of radiographs that were taken today. Yours sincerely, ____________ Dr ABC (BDS) Enc: bitewing and DPT radiographs ---------------------------------------------------------------------------------------------------------------------------------------

112

Denture faults Candidate instructions Proceed to examine these completed full upper and lower dentures Examine them on the models and hand articulated Identify seven errors in their construction What is the name given to the most common defect arising in the base of polymerised acrylic resin denture? Briefly outline how this defect occurs and what it may look like?

113

Fluoride therapy and age of child Dental panoramic tomography and intraoral photographs provided How old is the child? The child was having lower first premolars and lower canine so I said he is 10 to 11 years Which fluoride supplement you will give to the child, local or systemic The child was having lots o caries and also the water is not fluoridated so I chose systemic fluoride supplement How much fluoride supplement you will give? 1 mg of fluoride tablets Mention name of topical fluoride Fluoride varnish – Duraphat 2600ppm What two special investigations will carry out for this patient? Diet analysis and bitewings Questions: 1. Give dental age – 7-8 years 2. Caries risk assessment 3. Two investigations to be done 4. Would you consider this child for systemic Fluorides? Why? 5. Prescribe systemic fluorides (1mg Fluoride tablet per day if water has less than 0.3ppm) 6. Would you consider this child for pit and fissure sealants? Why? Finding dental age – Identify the teeth present on the OPG and follow the eruption chart.

114

115

Caries risk assessment Diet and sugar intake – (time, content and quantity of sugars including bottle or breast feeding in a day) - >3 = high risk, /= 3

Motivation of mother and family - regular attendee to dentist – low risk Socio-economic status Relevant medical history – high risk in medically compromised or children with special needs Tests to be done Diet Analysis Bite wing radiographs Reasons for prescribing fluoride supplements DMFT in moderate/high risk 1 to 4 = Medium, >5 or 5 = High At 6-8 years, caries in 1st permanent molar = High risk Number of initial lesions – None = Low risk, >10 = High risk Fluoride supplements Children at risk should be considered for fluoride supplements. Water content level Age of child 6 months – 3rd birthday

< 0.3 ppm

0.3–0.7 ppm

250 µg daily (need to halve 500 µg tablet )

Not advised

3–6th birthday

500 µg daily

250 µg daily (need to halve 500 µg tablet )

6 and over

1 mg daily

> 0.7 ppm

Not advised

500 µg daily

Indications for pit and fissure sealants Children with extensive caries in primary teeth should have all permanent molars sealed as soon as they erupt. Occlusal caries in any 1st permanent molar indicates need to seal all the permanent molars. Children with special needs, medically compromised, learning difficulty, physically and dentally compromised, socio-economically disadvantaged.

116

Prevention of caries in children aged 0–6 years Advice to be given

Professional intervention

Children aged up to 3 years

• Breast feeding provides the best nutrition for  babies • From six months of age infants should be  introduced to drinking from a cup, and from age one year feeding from a bottle should be discouraged • Sugar should not be added to weaning foods • Parents should brush or supervise  toothbrushing • Use only a smear of toothpaste containing no  less than 1,000 ppm fluoride • As soon as teeth erupt in the mouth brush them twice daily • The frequency and amount of sugary food and  drinks should be reduced and, when consumed, limited to mealtimes. Sugars should not be consumed more than four times per day • Sugar-free medicines should be recommended

All children aged 3–6 years

• Brush last thing at night and on one other  occasion • Brushing should be supervised by an adult • Use a pea-sized amount of toothpaste containing 1,350–1,500 ppm fluoride • Spit out after brushing and do not rinse • The frequency and amount of sugary food and drinks should be reduced and, when consumed, limited to mealtimes. Sugars should not be consumed more than four times per day • Sugar-free medicines should be recommended

• Apply fluoride varnish to teeth twice  yearly (2.2% F–)

Children giving concern (eg those likely to develop caries, those with special needs)

All advice as above, plus: • Use a smear or pea-sized amount of toothpaste containing 1,350–1,500 ppm fluoride • Ensure medication  is sugar free • Give dietary supplements containing sugar and  glucose polymers at mealtimes when possible (unless clinically directed otherwise) and not last thing at night. Parents should be made aware of the cariogenicity of supplements and ways of minimising risk

• Apply fluoride varnish to teeth 3–4 times yearly (2.2% F–) • Prescribe fluoride supplement and  advise re maximising benefit • Reduce recall interval • Investigate diet and assist to adopt  good dietary practice • Ensure medication  is sugar free or  given to minimise cariogenic effect

117

Prevention of caries in children aged from 7 years and young adults Advice

Professional intervention

All children and young adults

• Brush twice daily • Brush last thing at night and on one other  occasion • Use fluoridated toothpaste (1,350 ppm fluoride or above) • Spit out after brushing and do not rinse • The frequency and amount of sugary food and  drinks should be reduced and, when consumed, limited to mealtimes. Sugars should not be consumed more than four times per day

• Apply fluoride varnish to teeth  twice yearly (2.2% F–)

Those giving concern (eg those likely to develop caries, those undergoing orthodontic treatment, those with special needs)

All the above, plus: • Fissure seal permanent molars  • Use a fluoride mouth rinse daily (0.05% NaF) at  with resin sealant a different time to brushing • Apply fluoride varnish to teeth  3–4 times yearly (2.2% F–) • For those 8+ years with active  caries prescribe daily fluoride rinse • For those 10+ years with active  caries prescribe 2,800 ppm toothpaste • For those 16+ years with active  disease consider prescription of 5,000 ppm toothpaste • Investigate diet and assist  adoption of good dietary practice

118

Prevention of caries in adults Advice All adult patients

• Brush twice daily with fluoridated  toothpaste • Use fluoridated toothpaste with at least  1,350 ppm fluoride • Brush last thing at night and on one  other occasion • Spit out after brushing and do not rinse • The frequency and amount of sugary food and drinks should be reduced and, when consumed, limited to mealtimes. Sugars should not be consumed more than four times per day

Those giving concern to their dentist (eg with obvious current active caries, dry mouth, other predisposing factors, those with special needs)

All the above, plus: • Use a fluoride mouth rinse daily (0.05%  NaF) at a different time to brushing

Professional intervention

• Apply fluoride varnish to teeth twice  yearly (2.2% F–) • For those with obvious active coronal  or root caries prescribe daily fluoride rinse • For those with obvious active coronal  or root caries prescribe 2,800 or 5,000 ppm fluoride toothpaste • Investigate diet and assist adoption of  good dietary practice

119

Prevention of periodontal disease – to be used in addition to caries prevention Risk level Advice Professional intervention All adolescents and Adults

Children with difficulty maintaining oral hygiene; with relevant medical conditions; wearing orthodontic appliances

• Brush teeth systematically twice daily with  either: – a manual brush with a small head and round end filaments, a compact, angled arrangement of long and short filaments and a comfortable handle or – a powered toothbrush with an oscillating/rotating head • Do not smoke • Consider using toothpastes containing: – triclosan with copolymer, or – triclosan with zinc citrate to improve levels of plaque control • Toothpastes with stannous fluoride may reduce gingivitis • Clean interdentally using interdental brushes  or floss • Maintain good dietary practices in line with   The Balance of Good Health

• Demonstrate methods of  improving plaque Control

• Brush systematically twice daily with either: – a manual brush with a small head and round end filaments, a compact, angled arrangement of long and short filaments and a comfortable handle OR – a powered toothbrush with an oscillating/rotating head • Maintain good dietary practices

• Demonstrate methods of  improving plaque control • Investigate diet and assist  adoption of good dietary practice

• Investigate possible improved  control of predisposing systemic conditions • Take a history of tobacco use,  give brief advice to users and signpost to local Stop Smoking Service • Investigate diet and assist  adoption of good dietary practice based on The Balance of Good Health

Prevention of oral cancer Risk level All adolescents and adults

Advice

Professional intervention

• Do not smoke • Do not use smokeless tobacco (eg paan,  chewing tobacco, gutkha) • Reduce alcohol consumption  to moderate (recommended) levels • Maintain good dietary practices in line with  The Balance of Good Health • Increase fruit and vegetable intake to at least  five portions per day

• Take a history of tobacco use,  give brief advice to users and signpost to local Stop Smoking Service • Signpost to local alcohol  misuse support services

120

Prescribing high concentration fluoride toothpaste Sodium fluoride 2,800 ppm toothpaste Indications: high caries risk patients aged 10 years and over.

Sodium fluoride 5,000 ppm toothpaste Indications: patients aged 16 years and over with high caries risk, present or potential for root caries, dry mouth, orthodontic appliances, overdentures, those with highly cariogenic diet or medication.

Use of fluoride supplements – prescribing information It is recognised that the use of fluoride tablets requires compliance by families and this may include under and over-use. There is a risk of fluorosis if children aged under 6 years take over the advised dose. With this in mind, other sources of fluoride may be preferable and therefore be considered first. Tablets are available in 500 μg and 1 mg fluoride levels. Tablets should be given at a different time to tooth brushing and allowed to dissolve slowly in the mouth to maximise their topical effect.

121

Fluoride rinses These can be prescribed for patients aged 8 years and above, for daily or weekly use, in addition to twice daily brushing with toothpaste containing at least 1,350 ppm fluoride. Rinses require patient compliance and should be used at a different time to toothbrushing to maximise the topical effect, which relates to frequency of availability.

122

123